Clinical Cases in Dermatopathology (Clinical Cases in Dermatology) 3030288064, 9783030288068

This book provides a case-based guide to the diagnosis and management of dermatopathological conditions. Clinical cases

109 8 14MB

English Pages 190 [178] Year 2020

Report DMCA / Copyright

DOWNLOAD FILE

Polecaj historie

Clinical Cases in Dermatopathology (Clinical Cases in Dermatology)
 3030288064, 9783030288068

Table of contents :
Contents
Chapter 1: Case 1: Skin Lesions of Trunk and Extremities, Clinical R/O Scabies
History and Clinical
Physical Examination
Clinical Differential Diagnosis
Histopathology
Final Diagnosis
Case Discussion
Clinical Management
Suggested Reading
Chapter 2: Case 2: Leg Lesion with Clinical Impression: Stasis vs. Skin Cancer
History and Clinical
Physical Examination
Clinical Figures
Clinical Differential Diagnosis
Histopathology
Final Diagnosis
Case Discussion
Clinical Management
Suggested Reading
Chapter 3: Case 3: Skin Lesions of Trunk and Extremities, Clinical R/O Scabies
History and Clinical
Physical Examination
Clinical Differential Diagnosis
Histopathology
Final Diagnosis
Case Discussion
Clinical Management
Suggested Reading
Chapter 4: Case 4: Skin Lesion of Forehead, Clinical Impression, R/O Foreign Object
History and Clinical
Physical Examination
Clinical Differential Diagnosis
Histopathology
Final Diagnosis
Case Discussion
Clinical Management
Suggested Reading
Chapter 5: Case 5: Skin Lesions of Forehead, R/O Sebaceoma
History and Clinical
Physical Examination
Clinical Figures
Clinical Differential Diagnosis
Histopathology
Final Diagnosis
Pathology Discuss
Clinical Management
Suggested Reading
Chapter 6: Case 6: Skin Lesion of Right Lower Leg, R/O Vitiligo
History and Clinical
Physical Examination
Clinical Figures
Clinical Differential Diagnosis
Histopathology
Final Diagnosis
Case Discussion
Clinical Management
Suggested Reading
Chapter 7: Case 7: Skin of Scalp, R/O Tinea vs. Seborrheic Dermatitis
History and Clinical
Physical Examination
Clinical Figures
Clinical Differential Diagnosis
Histopathology
Final Diagnosis
Case Discussion
Clinical Management
Suggested Reading
Chapter 8: Case 8: Skin Lesion of Abdomen, Clinically Elevated
History and Clinical
Physical Examination
Clinical Differential Diagnosis
Histopathology
Final Diagnosis
Case Discussion
Clinical Management
Suggested Reading
Chapter 9: Case 9: Skin of Right Inferior Eyelid Lesion, R/O Seborrheic Keratosis
History and Clinical
Physical Examination
Clinical Figure(s)
Clinical Differential Diagnosis
Histopathology
Final Diagnosis
Case Discussion
Clinical Management
Suggested Reading
Chapter 10: Case 10: Four-Year-Old Female with Left Lateral Trunk Lesions
History and Clinical
Physical Examination
Clinical Figures
Clinical Differential Diagnosis
Histopathology
Final Diagnosis
Case Discussion
Clinical Management
Suggested Reading
Chapter 11: Case 11: Skin Lesion Left Upper Arm, R/O Cyst
History and Clinical
Physical Examination
Clinical Figure(s)
Clinical Differential Diagnosis
Histopathology
Final Diagnosis
Case Discussion
Clinical Management
Suggested Reading
Chapter 12: Case 12: Skin of Back, R/O Cancer vs. Dermatitis, vs. Tinea
History and Clinical
Physical Examination
Clinical Figures
Clinical Differential Diagnosis
Histology and Diagnosis
Histopathology
Final Diagnosis
Case Discussion
Clinical Management
Suggested Reading
Chapter 13: Case 13: Skin of Right Wrist, R/O Foreign Object
History and Clinical
Physical Examination
Clinical Figure(s)
Clinical Differential Diagnosis
Histopathology
Final Diagnosis
Case Discussion
Clinical Management
Suggested Reading
Chapter 14: Case 14: Left Parietal Skin, R/O Dysplastic Nevus
History and Clinical
Physical Examination
Clinical Figure(s)
Clinical Differential Diagnosis
Histopathology
Final Diagnosis
Case Discussion
Clinical Management
Suggested Reading
Chapter 15: Case 15: Skin of Mid Lateral Back, R/O Fibroma
History and Clinical
Physical Examination
Clinical Figure(s)
Clinical Differential Diagnosis
Histopathology
Final Diagnosis
Case Discussion
Clinical Management
Suggested Reading
Chapter 16: Case 16: Skin of Upper Abdomen, R/O Squamous Cell Carcinoma vs. Basal Cell Carcinoma
History and Clinical
Physical Examination
Clinical Figure(s)
Clinical Differential Diagnosis
Histopathology
Final Diagnosis
Case Discussion
Clinical Management
Suggested Reading
Chapter 17: Case 17: Skin of Upper Paraspinal Lesion, R/O Miliaria Rubra vs. Atopic Dermatitis
History and Clinical
Physical Examination
Clinical Differential Diagnosis
Histopathology
Final Diagnosis
Case Discussion
Clinical Management
Suggested Reading
Chapter 18: Case 18: Skin Right Forearm, R/O Rhus Derm
History and Clinical
Physical Examination
Clinical Figure(s)
Clinical Differential Diagnosis
Histopathology
Diagnosis
Case Discussion
Clinical Management
Suggested Reading
Chapter 19: Case 19: Left Mandible Skin, R/O Basal Cell Carcinoma vs. Cyst
History and Clinical
Physical Examination
Clinical Differential Diagnosis
Histology and Diagnosis
Histopathology
Diagnosis
Case Discussion
Clinical Management
Suggested Reading
Chapter 20: Case 20: Occipital Scalp Skin, R/O Squamous Cell Carcinoma
History and Clinical
Physical Examination
Clinical Figures
Clinical Differential Diagnosis
Histopathology
Final Diagnosis
Case Discussion
Clinical Management
Suggested Reading
Chapter 21: Case 21: Skin from Mid-Forehead, R/O Verruca Vulgaris
History and Clinical
Physical Examination
Clinical Figure(s)
Clinical Differential Diagnosis
Histopathology
Final Diagnosis
Pathology Discussion
Clinical Management
Suggested Reading
Chapter 22: Case 22: Right Upper Arm Lesion
History and Clinical
Physical Examination
Clinical Differential Diagnosis
Histopathology and Diagnosis
Histopathology
Final Diagnosis
Case Discussion
Clinical Management
Suggested Reading
Chapter 23: Case 23: Right Arm Papules
History and Clinical
Physical Examination
Clinical Figures
Clinical Differential Diagnosis
Histopathology
Final Diagnosis
Case Discussion
Clinical Management
Suggested Reading
Chapter 24: Case 24: Nodular Masses on Left Fingertips
History and Clinical
Physical Examination
Clinical Figure(s)
Clinical Differential Diagnosis
Histopathology
Final Diagnosis
Case Discussion
Suggested Reading
Chapter 25: Case 25: Diffuse Rashes Across Back, Thighs, and Legs
History and Clinical
Physical Examination
Clinical Figures
Clinical Differential Diagnosis
Histopathology and Diagnosis
Histopathology
Final Diagnosis
Pathology Discussion
Clinical Management
Suggested Reading
Index

Citation preview

Clinical Cases in Dermatology Series Editor: Robert A. Norman

Dong-Lin Xie Editor

Clinical Cases in Dermatopathology

Clinical Cases in Dermatology Series Editor Robert A. Norman Tampa, FL, USA

This series of concise practical guides is designed to facilitate the clinical decision-making process by reviewing a number of cases and defining the various diagnostic and management decisions open to clinicians. Each title is illustrated and diverse in scope, enabling the reader to obtain relevant clinical information regarding both standard and unusual cases in a rapid, easy to digest format. Each focuses on one disease or patient group, and includes common cases to allow readers to know they are doing things right if they follow the case guidelines. More information about this series at http://www.springer. com/series/10473

Dong-Lin Xie Editor

Clinical Cases in Dermatopathology

Editor

Dong-Lin Xie Ruffolo, Hooper & Associates, M.D, P.A. Tampa, FL USA

Clinical Cases in Dermatology ISBN 978-3-030-28806-8    ISBN 978-3-030-28807-5 (eBook) https://doi.org/10.1007/978-3-030-28807-5 © Springer Nature Switzerland AG 2020 This work is subject to copyright. All rights are reserved by the Publisher, whether the whole or part of the material is concerned, specifically the rights of translation, reprinting, reuse of illustrations, recitation, broadcasting, reproduction on microfilms or in any other physical way, and transmission or information storage and retrieval, electronic adaptation, computer software, or by similar or dissimilar methodology now known or hereafter developed. The use of general descriptive names, registered names, trademarks, service marks, etc. in this publication does not imply, even in the absence of a specific statement, that such names are exempt from the relevant protective laws and regulations and therefore free for general use. The publisher, the authors, and the editors are safe to assume that the advice and information in this book are believed to be true and accurate at the date of publication. Neither the publisher nor the authors or the editors give a warranty, expressed or implied, with respect to the material contained herein or for any errors or omissions that may have been made. The publisher remains neutral with regard to jurisdictional claims in published maps and institutional affiliations. This Springer imprint is published by the registered company Springer Nature Switzerland AG The registered company address is: Gewerbestrasse 11, 6330 Cham, Switzerland

Contents

1 Case 1: Skin Lesions of Trunk and Extremities, Clinical R/O Scabies�������������������������������������������������������   1 Dong-Lin Xie, Tam H. Nguyen, and Robert A. Norman 2 Case 2: Leg Lesion with Clinical Impression: Stasis vs. Skin Cancer�����������������������������������������������������   7 Dong-Lin Xie, Tam H. Nguyen, and Robert A. Norman 3 Case 3: Skin Lesions of Trunk and Extremities, Clinical R/O Scabies�������������������������������������������������������  15 Dong-Lin Xie, Tam H. Nguyen, and Robert A. Norman 4 Case 4: Skin Lesion of Forehead, Clinical Impression, R/O Foreign Object �������������������  19 Dong-Lin Xie, Tam H. Nguyen, and Robert A. Norman 5 Case 5: Skin Lesions of Forehead, R/O Sebaceoma �������������������������������������������������������������  25 Dong-Lin Xie, Tam H. Nguyen, and Robert A. Norman 6 Case 6: Skin Lesion of Right Lower Leg, R/O Vitiligo���������������������������������������������������������������������  35 Dong-Lin Xie, Tam H. Nguyen, and Robert A. Norman 7 Case 7: Skin of Scalp, R/O Tinea vs. Seborrheic Dermatitis ���������������������������������������������������  41 Dong-Lin Xie, Tam H. Nguyen, and Robert A. Norman 8 Case 8: Skin Lesion of Abdomen, Clinically Elevated���������������������������������������������������������  49 Dong-Lin Xie, Tam H. Nguyen, and Robert A. Norman

vi

Contents

9 Case 9: Skin of Right Inferior Eyelid Lesion, R/O Seborrheic Keratosis���������������������������������������������  55 Dong-Lin Xie, Tam H. Nguyen, and Robert A. Norman 10 Case 10: Four-Year-Old Female with Left Lateral Trunk Lesions ���������������������������������������������������  61 Dong-Lin Xie, Tam H. Nguyen, and Robert A. Norman 11 Case 11: Skin Lesion Left Upper Arm, R/O Cyst�������  69 Dong-Lin Xie, Tam H. Nguyen, and Robert A. Norman 12 Case 12: Skin of Back, R/O Cancer vs. Dermatitis, vs. Tinea�������������������������������������������������������  75 Dong-Lin Xie, Tam H. Nguyen, and Robert A. Norman 13 Case 13: Skin of Right Wrist, R/O Foreign Object�������������������������������������������������������  83 Dong-Lin Xie, Tam H. Nguyen, and Robert A. Norman 14 Case 14: Left Parietal Skin, R/O Dysplastic Nevus ���������������������������������������������������  89 Dong-Lin Xie, Tam H. Nguyen, and Robert A. Norman 15 Case 15: Skin of Mid Lateral Back, R/O Fibroma�������������������������������������������������������������������  97 Dong-Lin Xie, Tam H. Nguyen, and Robert A. Norman 16 Case 16: Skin of Upper Abdomen, R/O Squamous Cell Carcinoma vs. Basal Cell Carcinoma����������������������������������������������������� 103 Dong-Lin Xie, Tam H. Nguyen, and Robert A. Norman 17 Case 17: Skin of Upper Paraspinal Lesion, R/O Miliaria Rubra vs. Atopic Dermatitis ����������������� 109 Dong-Lin Xie, Tam H. Nguyen, and Robert A. Norman 18 Case 18: Skin Right Forearm, R/O Rhus Derm ��������� 115 Dong-Lin Xie, Tam H. Nguyen, and Robert A. Norman 19 Case 19: Left Mandible Skin, R/O Basal Cell Carcinoma vs. Cyst������������������������������������������������� 121 Dong-Lin Xie, Tam H. Nguyen, and Robert A. Norman

Contents

vii

20 Case 20: Occipital Scalp Skin, R/O Squamous Cell Carcinoma��������������������������������������������������������������� 129 Dong-Lin Xie, Tam H. Nguyen, and Robert A. Norman 21 Case 21: Skin from Mid-­Forehead, R/O Verruca Vulgaris����������������������������������������������������� 137 Dong-Lin Xie, Tam H. Nguyen, and Robert A. Norman 22 Case 22: Right Upper Arm Lesion������������������������������� 143 Dong-Lin Xie, Emelia Farnsworth, and Robert A. Norman 23 Case 23: Right Arm Papules����������������������������������������� 151 Dong-Lin Xie, Emelia Farnsworth, and Robert A. Norman 24 Case 24: Nodular Masses on Left Fingertips��������������� 163 Dong-Lin Xie, Emelia Farnsworth, and Robert A. Norman 25 Case 25: Diffuse Rashes Across Back, Thighs, and Legs��������������������������������������������������������������������������� 171 Dong-Lin Xie, Emelia Farnsworth, and Robert A. Norman Index����������������������������������������������������������������������������������������� 179

Chapter 1 Case 1: Skin Lesions of Trunk and Extremities, Clinical R/O Scabies Dong-Lin Xie, Tam H. Nguyen, and Robert A. Norman

History and Clinical A 92-year-old male with a history of hemiplegia and hemiparesis following a cerebrovascular accident presented in the nursing home with a widespread rash for several weeks. Nursing home staff noted the patient was scratching ­throughout the day, but it would become significantly worse during the night. The patient had no history of similar skin complaints.

D.-L. Xie (*) Ruffolo, Hooper & Associates, M.D, P.A., Tampa, FL, USA e-mail: [email protected] T. H. Nguyen DO, PGY-1 Larkin Community Hospital, Miami, FL, USA R. A. Norman Department of Dermatology, Nova Southeastern University, Tampa, FL, USA Center for Geriatric Dermatology and Neuro-Dermatology, Tampa, FL, USA © Springer Nature Switzerland AG 2020 D.-L. Xie (ed.), Clinical Cases in Dermatopathology, Clinical Cases in Dermatology, https://doi.org/10.1007/978-3-030-28807-5_1

1

2

D.-L. Xie et al.

Physical Examination The patient had a diffuse, erythematous papular rash on his trunk and groin. Overlying areas of linear excoriations were noted as well on the trunk and folds of the groin. Very discrete linear burrows were also seen between the interdigital spaces of his right and left hands.

Clinical Differential Diagnosis • • • • •

Insect bites Atopic Dermatitis Neurotic Excoriations Scabies Drug Reaction

Histopathology Spongiotic epidermal changes with marked mixed perivascular and interstitial inflammatory cells of lymphocytes, histiocytes, abundant eosinophils and occasional neutrophils are present (Fig. 1.1). Hyperkeratosis and parakeratosis with neutrophils are seen in the stratum corneum. Organisms and material morphologically consistent with scabies and scabies products are identified (Figs. 1.2 and 1.3). In addition, PAS stain reveals fungal hyphae in the in the stratum corneum (Fig. 1.4).

Final Diagnosis –– Scabies infestation. –– Dermatophytosis.

Chapter 1.  Case 1: Skin Lesions of Trunk…

3

Figure 1.1 Spongiosis and mixed inflammation with frequent eosinophils

Figure 1.2  Mature form of scabies

4

D.-L. Xie et al.

Figure 1.3  Scabies fecal material

Figure 1.4  PAS stain positive for fungal hyphae in the stratum corneum

Chapter 1.  Case 1: Skin Lesions of Trunk…

5

Case Discussion • Scabies can be focal in histologic sections. If you did not find scabies and its products in the initial sections, multiple level sections should be performed to improve the yield of positive findings. Failure to find scabies and its products do not completely exclude scabies infestation. • Skin rash/dermatitis may have more than one etiology. Co-­ infection and infestation may present in the same patient. When neutrophils are present in the stratum corneum or the stratum corneum is significantly thickened, PAS or GMS stain to exclude fungal infection should be considered.

Clinical Management This patient was treated for scabies based upon histopathological diagnosis with oral Ivermectin and loratadine to control pruritus. Follow up 4  weeks later revealed significant improvement.

Suggested Reading Camacaro L, Reales E. Scabies. J Am Acad Dermatol. 2015;72(5):1, AB129. Suwandhi P, Dharmarajan TS.  Scabies in the nursing home. Curr Infect Dis Rep. 2015;17(1):453. Tarbox M, Walker K, Tan M. Scabies. JAMA. 2018;320(6):612. Woloski JR, Burman D, Adebona O.  Mite and bed bug infections. Prim Care. 2018;45(3):409–21.

Chapter 2 Case 2: Leg Lesion with Clinical Impression: Stasis vs. Skin Cancer Dong-Lin Xie, Tam H. Nguyen, and Robert A. Norman

History and Clinical A 76-year-old male nursing home patient with a history of dementia was evaluated for a red crusted area on his left lower extremity. He had a previous history of neurotic excoriations and lichen simplex chronicus for which he was being treated with mupirocin 2% ointment, and triamcinolone cream. The lesion on the left lower extremity had been present for several years, and was asymptomatic, but recently became more red and scaly. The patient denied any pain or pruritus in the area.

D.-L. Xie (*) Ruffolo, Hooper & Associates, M.D, P.A., Tampa, FL, USA e-mail: [email protected] T. H. Nguyen DO, PGY-1 Larkin Community Hospital, Miami, FL, USA R. A. Norman Department of Dermatology, Nova Southeastern University, Tampa, FL, USA Center for Geriatric Dermatology and Neuro-Dermatology, Tampa, FL, USA © Springer Nature Switzerland AG 2020 D.-L. Xie (ed.), Clinical Cases in Dermatopathology, Clinical Cases in Dermatology, https://doi.org/10.1007/978-3-030-28807-5_2

7

8

D.-L. Xie et al.

Physical Examination The patient had a 9  cm by 5  cm, poorly demarcated, erythematous plaque with overlying crust, thick scale, and erosions on the left lower extremity (Figs. 2.1 and 2.2).

Figure 2.1  Poorly demarcated, erythematous plaque with overlying crust

Figure 2.2  Poorly demarcated, erythematous plaque with overlying crust, closer view

Chapter 2.  Case 2: Leg Lesion with Clinical Impression…

9

Clinical Figures Clinical Differential Diagnosis • • • •

Squamous cell carcinoma Lichen simplex chronicus, with irritation Bowen’s Disease Superficial Basal Cell Carcinoma

Histopathology Ulcerated lesion with dense diffuse predominantly mononucleated cell infiltrate (Fig. 2.3, low magnification). Most of the mononucleated cells are plasmacytoid under high magnification (Fig. 2.4, high magnification). Immunohistochemical studies (IHC) reveal that most of the mononucleated cells are positive for CD138 (Fig.  2.5,

Figure 2.3  Low magnification: Ulcerated lesion with dense diffuse mononucleated cell infiltrate

10

D.-L. Xie et al.

Figure 2.4  High magnification: Most of the mononucleated cells are plasma cells

Figure 2.5  IHC: CD138 are positive in tumor cells

Chapter 2.  Case 2: Leg Lesion with Clinical Impression…

11

Figure 2.6  Kappa ISH: overwhelming majority of plasma cells are kappa positive

IHC of CD138) with very few CD3 and CD20 positive cells, which confirm that the mononucleated cells are mostly plasma cells. In situ hybridization (ISH) with kappa and lambda tests demonstrate that the kappa to lambda ratio is greater than 12:1 (Figs.  2.6 and 2.7). Such high ratio is consistent with a clonal plasma cell proliferation with kappa restriction.

Final Diagnosis –– Plasmacytoma/multiple myeloma.

Case Discussion • Plasmacytoma involving skin clinically can mimic other skin cancers. • Most cases of dense plasma cell infiltrate in skin are reactive or associate with infection. In an elderly patient, if the

12

D.-L. Xie et al.

Figure 2.7  Lambda ISH: Small numbers of plasma cells are lambda positive

skin lesion has dense plasma cell infiltrate, the pathologist should consider the possibility of plasmacytoma or multiple myeloma involving skin. Additional studies to evaluate kappa to lambda ratio would help to clarify reactive plasma cell infiltration vs. plasmacytoma/multiple myeloma. • Plasmacytoma may co-exist with other skin cancer. • The term “Plasmacytoma” applies to localized lesions after excluding systemic involvement (peripheral blood and bone marrow), i.e. after excluding presence of multiple myeloma through additional studies.

Clinical Management The patient was referred to a cancer center for evaluation and treatment of the plasmacytoma.

Chapter 2.  Case 2: Leg Lesion with Clinical Impression…

13

Suggested Reading Agarwal SC. Extramedullary plasmacytoma; report of a case. AMA Arch Derm. 1956;74(6):679–80. Fuji H, Maekawa T, Kanoh T, Ohnaka T, Nishida K.  An autopsy case of primary cutaneous plasmacytoma. Rinsho Ketsueki. 1989;30(2):256–61. Torne R, Su WP, Winkelmann RK, Smolle J, Kerl H. Clinicopathologic study of cutaneous plasmacytoma. Int J Dermatol. 1990;29(8):562–6. Zhang S, Li WH, Zhao Y, Cai L. A case report of cutaneous plasmacytosis. Beijing Da Xue Xue Bao. 2018;50(4):752–4.

Chapter 3 Case 3: Skin Lesions of Trunk and Extremities, Clinical R/O Scabies Dong-Lin Xie, Tam H. Nguyen, and Robert A. Norman

History and Clinical An 82-year-old female was seen in a nursing home for a widespread rash on the abdomen, lower extremities, groin folds, and buttocks. The rash began 2 months ago and was extremely itchy, especially at night time. She had no previous history of any skin conditions.

Physical Examination The patient had a widespread erythematous papular rash on the chest, abdomen, lower extremities, and in the intertrigi-

D.-L. Xie (*) Ruffolo, Hooper & Associates, M.D, P.A., Tampa, FL, USA e-mail: [email protected] T. H. Nguyen DO, PGY-1 Larkin Community Hospital, Miami, FL, USA R. A. Norman Department of Dermatology, Nova Southeastern University, Tampa, FL, USA Center for Geriatric Dermatology and Neuro-Dermatology, Tampa, FL, USA © Springer Nature Switzerland AG 2020 D.-L. Xie (ed.), Clinical Cases in Dermatopathology, Clinical Cases in Dermatology, https://doi.org/10.1007/978-3-030-28807-5_3

15

16

D.-L. Xie et al.

nous areas of the groin. Similar lesions were seen between the gluteal folds as well.

Clinical Differential Diagnosis • • • • •

Candidiasis Atopic Dermatitis Insect bites Scabies Drug Reaction

Histopathology Spongiotic epidermal changes with marked mixed perivascular and interstitial inflammatory cells of lymphocytes, histiocytes, abundant eosinophils and occasional neutrophils are present, as well as all forms of scabies and its products in the stratum corneum in the same area (Fig. 3.1, low magnification).

Figure 3.1  Scabies and its products, under low magnification

Chapter 3.  Case 3: Skin Lesions of Trunk and Extremities…

17

Figure 3.2  One mature, two ova and fecal material of scabies, under high magnification

Figure 3.3  One ovum and abundant fecal material, under high magnification

18

D.-L. Xie et al.

Under high magnifications (Figs. 3.2 and 3.3), one can find mature scabies, scabies ova and fecal material.

Final Diagnosis –– Scabies infestation.

Case Discussion • This case does not have diagnostic difficulty. The histopathological features are textbook classical. Most of the time in a clinically suspected scabies case, pathologists have to request additional level sections to identify scabies organisms or its products. • What is special about this case is one can find mature scabies, multiple scabies ova and fecal material in the same microscopic field.

Clinical Management Based on histopathological and clinical correlation, the patient was treated for scabies with oral Ivermectin, fluocinonide 0.05% ointment, and loratadine.

Suggested Reading Camacaro L, Reales E. Scabies. J Am Acad Dermatol. 2015;72(5):1, AB129. Modi K, Patel D, Shwayder T.  Scalp-to-toes application of permethrin for patients with scabies. Dermatol Online J. 2018;24:5. Suwandhi P, Dharmarajan TS.  Scabies in the nursing home. Curr Infect Dis Rep. 2015;17(1):453. Tarbox M, Walker K, Tan M. Scabies. JAMA. 2018;320(6):612.

Chapter 4 Case 4: Skin Lesion of Forehead, Clinical Impression, R/O Foreign Object Dong-Lin Xie, Tam H. Nguyen, and Robert A. Norman

History and Clinical A 52-year-old man with a history of HIV presented with lumps on his forehead and temples for three weeks. He denied any pain or itching. He noticed these lumps first appeared three to four days after he applied a new “antiaging” cream containing retinol onto his face. He stated he only applied the cream to the forehead and temples. He has since stopped using this cream. He has a history of Botox

D.-L. Xie (*) Ruffolo, Hooper & Associates, M.D, P.A., Tampa, FL, USA e-mail: [email protected] T. H. Nguyen DO, PGY-1 Larkin Community Hospital, Miami, FL, USA R. A. Norman Department of Dermatology, Nova Southeastern University, Tampa, FL, USA Center for Geriatric Dermatology and Neuro-Dermatology, Tampa, FL, USA © Springer Nature Switzerland AG 2020 D.-L. Xie (ed.), Clinical Cases in Dermatopathology, Clinical Cases in Dermatology, https://doi.org/10.1007/978-3-030-28807-5_4

19

20

D.-L. Xie et al.

injections to the forehead and temple area as well, with his last treatment being several months ago.

Physical Examination The patient had discrete, erythematous macules and papules on the forehead and temporal regions (Fig. 4.1). There were no weeping, erosions, crusts or other associated skin changes.

Clinical Differential Diagnosis • • • • •

Allergic Contact Dermatitis Irritant Contact Dermatitis Granulomatous response to foreign objects (Botox) Atopic Dermatitis Seborrheic Dermatitis

Figure 4.1  Erythematous macules and papules on the forehead and temporal regions

Chapter 4.  Case 4: Skin Lesion of Forehead, Clinical…

21

Histopathology Dense dermal mixed inflammatory infiltrate of lymphocytes, histiocytes and multinucleated giant cells with variable sized vacuoles, “Swiss cheese”- like holes, is present (Figs. 4.2 and 4.3, under medium and high magnification). No polarizable foreign material is identified. AFB and GMS stains are negative for microorganisms (Fig.  4.4 GMS stain and Fig. 4.5 AFB stain).

Final Diagnosis –– Granulomatous inflammation, favoring Botox injection.

Case Discussion • Granulomatous reaction can be seen in infection, reaction to foreign material, sarcoidosis, ruptured folliculitis and other conditions.

Figure 4.2  “Swiss cheese”-like vacuoles, under medium magnification

22

D.-L. Xie et al.

Figure 4.3 Variable sized “Swiss cheese”-like vacuoles, under high magnification

Figure 4.4  GMS stain negative for fungal organisms

Chapter 4.  Case 4: Skin Lesion of Forehead, Clinical…

23

Figure 4.5  AFB stain negative for mycobacteria

• Not all foreign body granulomatous inflammation is polarizable. Some of the cosmetic and oil material injunction may not be polarizable. Granulomatous inflammation with Swiss cheese”- like vacuoles is suggestive of foreign material injection, which can be seen in cosmetic or percutaneous drug/medication injection. • Special stains to exclude infection should be performed. If the special stains are negative, and infection is a clinical consideration, additional tissue for microbiological culture should be considered.

Clinical Management Based on histopathological and clinical correlation, no treatment was advised, and the patient was advised to follow up as needed.

24

D.-L. Xie et al.

Suggested Reading Aoshima T, Oguchi M. Intracytoplasmic crystalline inclusions in dermal infiltrating cells of granulomatous contact dermatitis due to gold earrings. Acta Derm Venereol. 1988;68(3):261–4. Bhawan J. Steroid-induced ‘granulomas’ in hypertrophic scar. Acta Derm Venereol. 1983;63(6):560–3. Okada S, Okuyama R, Tagami H, Aiba S. Eosinophilic granulomatous reaction after intradermal injection of hyaluronic acid. Acta Derm Venereol. 2008;88(1):69–70. Takayama K, Satoh T, Yokozeki H.  Papular granuloma annulare with subcutaneous granulomatous reaction induced by a bee sting. Acta Derm Venereol. 2008;88(5):519–20.

Chapter 5 Case 5: Skin Lesions of Forehead, R/O Sebaceoma Dong-Lin Xie, Tam H. Nguyen, and Robert A. Norman

History and Clinical A 70-year-old man with a history of multiple skin cancers, including squamous cell carcinoma, and a history of sebaceous lesions presents for evaluation of several “moles” on his back, chest, and abdomen. These lesions were present for several years, but the patient wanted them evaluated given his history of skin cancer. He denied any bleeding, itching, or pain from the lesions. The patient denied any family history of cancer syndromes or colon cancer.

D.-L. Xie (*) Ruffolo, Hooper & Associates, M.D, P.A., Tampa, FL, USA e-mail: [email protected] T. H. Nguyen DO, PGY-1 Larkin Community Hospital, Miami, FL, USA R. A. Norman Department of Dermatology, Nova Southeastern University, Tampa, FL, USA Center for Geriatric Dermatology and Neuro-Dermatology, Tampa, FL, USA © Springer Nature Switzerland AG 2020 D.-L. Xie (ed.), Clinical Cases in Dermatopathology, Clinical Cases in Dermatology, https://doi.org/10.1007/978-3-030-28807-5_5

25

26

D.-L. Xie et al.

Physical Examination The patient had multiple, discrete, some skin colored, and some orange to red, papules on the right lateral mid and lower back (Figs. 5.1 and 5.2) and forehead (Fig. 5.3). Figure 5.1  Variable sized lesions on the lower back

Figure 5.2  Additional lesions on the lower back

Chapter 5.  Case 5: Skin Lesions of Forehead, R/O…

27

Figure 5.3  Erythematous and scaly lesions on the forehead

Clinical Figures Clinical Differential Diagnosis • • • • •

Sebaceoma Sebaceous carcinoma Nodular basal cell carcinoma Xanthoma Sebaceous adenoma

Histopathology This patient has multiple lesions and was biopsied at various time and had surgical resections. Most of the lesions are sebaceous neoplasms ranging from sebaceous hyperplasia to sebaceous carcinoma as well as squamous cell carcinoma.

28

D.-L. Xie et al.

Figure 5.4  Sebaceous hyperplasia with mature sebaceous glands

Sebaceous hyperplasia consists of proliferation of essentially mature sebaceous glands with one or two basaloid layers at the periphery of the glands (Fig. 5.4). Sebaceous adenoma has both basaloid layers and mature sebocytes, but with increased basaloid layers, also known as germinative layers (Fig.  5.5). However, the basaloid layer cells do not out number mature sebocytes. Sebaceoma, also known as sebaceous epithelioma presents as basaloid epithelioid proliferation with abrupt sebocytes differentiation (Figs. 5.6 and 5.7). The basaloid cells do out-­ number mature sebocytes. Patient may have different type of sebaceous lesions next to each other (Fig. 5.7). Sebaceous carcinomas most of the time have high grade malignant cytologic features. However, some sebaceous carcinomas may have more mature cytologic differentiation but with infiltrative pattern and abundant tumor necrosis, as seen in this case in this patient (Figs. 5.8 and 5.9).

Chapter 5.  Case 5: Skin Lesions of Forehead, R/O…

29

Figure 5.5  Sebaceous adenoma with increased germinative /basal layers

Figure 5.6  Sebaceoma, basaloid cells outnumber mature sebocytes

30

D.-L. Xie et al.

Figure 5.7 Sebaceous hyperplasia (left) adjacent to sebaceoma (right)

Figure 5.8  Sebaceous carcinoma with less mature (upper left) and more mature (low half of the picture) differentiations

Chapter 5.  Case 5: Skin Lesions of Forehead, R/O…

31

Figure 5.9  Sebaceous carcinoma with extensive necrosis

This patient also has other malignancy, squamous cell carcinoma, a proliferation of malignant squamous epithelium with marked nuclear atypia (Fig. 5.10). The above selective histologic figures are from variable sites of the biopsied and excised lesions.

Final Diagnosis –– –– –– –– ––

Sebaceous hyperplasia. Sebaceous adenoma. Sebaceoma. Sebaceous carcinoma. Invasive squamous cell carcinoma.

Pathology Discuss • Multiple sebaceous neoplasms have been associated with Muir-Torre syndrome in some patients. This syndrome has coexistence of multiple sebaceous tumors and multiple

32

D.-L. Xie et al.

Figure 5.10  Invasive squamous cell carcinoma

visceral carcinomas. The most common visceral carcinoma is colorectal carcinoma. • Patients may also have other type of skin carcinoma, mostly well-differentiated squamous cell carcinoma, including keratoacanthoma type. • Some of the sporadic sebaceoma can be predominantly basaloid cells. Basal cell carcinoma should be excluded as the differential diagnosis. Immuno studies with BerEP4 and CAM 5.2 may help to differentiate Basal cell carcinoma from sebaceoma.

Clinical Management Based on histopathological and clinical correlation and given the patient’s previous history of multiple similar sebaceous tumors and squamous cell carcinomas, the hereditary cancer syndrome Muir-Torre Syndrome was suspected. Muir-Torre Syndrome is an autosomal dominant cancer syndrome, con-

Chapter 5.  Case 5: Skin Lesions of Forehead, R/O…

33

sidered to be a subtype of Hereditary Nonpolyposis Colorectal Cancer (Lynch) Syndrome. This condition is due to mutations in DNA mismatch repair genes, including MLH1 and MSH2. Patients often have a predilection for cutaneous sebaceous tumors, most commonly sebaceous adenomas. They may also get sebaceous epitheliomas and sebaceous carcinomas. The eyelid is a very common area of involvement of sebaceous carcinomas. Keratoacanthomas are also seen in this syndrome. Patients with Muir-Torre syndrome can have increased risks of internal malignancies, most commonly colorectal carcinoma, endometrial carcinoma, breast cancer and ovarian serous cystadenocarcinoma. This patient’s pathology diagnoses were discussed with his primary care physician and recommendations for more vigilant cancer screening were prescribed.

Suggested Reading Brown S, Brennan P, Rajan N. Inherited skin tumour syndromes. Clin Med (Lond). 2017;17(6):562–7. Flux K. Sebaceous neoplasms. Surg Pathol Clin. 2017;97(8):959–61. Knackstedt T, Samie FH. Sebaceous carcinoma: a review of the scientific literature. Curr Treat Options in Oncol. 2017;18(8):47. Lacobelli J, Harvey NT, Wood BA.  Sebaceous lesions of the skin. Pathology. 2017;49(7):688–97. Le S, Ansari U, Mumtaz A, Malik K, Patel P, Doyle A, Khachemoune A.  Lynch syndrome and Muir-Torre syndrome: an update and review on the genetics, epidemiology, and management of two related disorders. Dermatol Online J. 2017;23:11. Pollinger TH, Kieliszak CR, Logemann N, Gratrix ML. Analysis of sebaceous neoplasms for DNA mismatch repair proteins in Muir-­ Torre syndrome. Skinmed. 2017;15(4):259–64.

Chapter 6 Case 6: Skin Lesion of Right Lower Leg, R/O Vitiligo Dong-Lin Xie, Tam H. Nguyen, and Robert A. Norman

History and Clinical A 58-year-old female with no significant past medical history presented with complain of skin pigment changes at various parts of the body for the past several months. She reported some associated itching at these sites.

D.-L. Xie (*) Ruffolo, Hooper & Associates, M.D, P.A., Tampa, FL, USA e-mail: [email protected] T. H. Nguyen DO, PGY-1 Larkin Community Hospital, Miami, FL, USA R. A. Norman Department of Dermatology, Nova Southeastern University, Tampa, FL, USA Center for Geriatric Dermatology and Neuro-Dermatology, Tampa, FL, USA © Springer Nature Switzerland AG 2020 D.-L. Xie (ed.), Clinical Cases in Dermatopathology, Clinical Cases in Dermatology, https://doi.org/10.1007/978-3-030-28807-5_6

35

36

D.-L. Xie et al.

Physical Examination The patient had discrete, well-demarcated, annular, 1–5 mm hypopigmented macules and patches at the upper and lower extremities (Fig. 6.1). There were no signs of xerosis or other cutaneous changes.

Clinical Figures Clinical Differential Diagnosis • • • • •

Vitiligo Idiopathic guttate hypomelanosis Pityriasis Alba Post inflammatory hypopigmentation Bier spots

Figure 6.1  Hypopigmented macules

Chapter 6.  Case 6: Skin Lesion of Right Lower Leg…

37

Histopathology The biopsy sections reveal minimal inflammatory cells with slightly thickened stratum corneum and subtle difference of pigmentation (Fig.  6.2). PAS stain is negative for fungal organisms (Fig.  6.3). Fontana-Masson stain reveals half of basilar keratinocytes having no melanin pigmentation (Fig.  6.4). Mart-1 stain reveals no junctional melanocytes (Fig.  6.5) in the same area of basilar keratinocytes lack of melanin pigmentation.

Final Diagnosis –– Vitiligo.

Figure 6.2  Minimal inflammation and subtle variation of pigmentation in the basal keratinocytes

38

D.-L. Xie et al.

Figure 6.3  Negative PAS for Fungal organisms

Figure 6.4  Fontana stain: lack of melanin pigmentation in the left half of basal keratinocytes

Chapter 6.  Case 6: Skin Lesion of Right Lower Leg…

39

Figure 6.5  Mart-1 Immunostain reveals lack of melanocytes in the left half of basal keratinocytes

Case Discussion • This biopsy is from the border of the lesional and normal skin. The histologic section is subtle in the differences of pigmentation in basal layer keratinocytes. • Loss of pigmentation can be seen in fungal infection. PAS or GMS to exclude fungal infection should be considered. Fungal infection may or may not induce significant inflammation. • Fontana and Mart-1 stain demonstrate loss of pigmentation and loss of normal junctional melanocytes, respectively. The reason of performing both studies is that some of cases, there are significant loss of pigmentation without corresponding loss of junctional melanocytes. In the latter situation, decreased melanin pigmentation could be temporary, as long as the function of the melanocytes is intact. • Decrease of melanin pigmentation may happen in conditions that are not vitiligo. Trauma and scar are some of the examples.

40

D.-L. Xie et al.

Clinical Management Based on histopathological and clinical correlation, a diagnosis of vitiligo was made. The patient was prescribed triamcinolone acetonide cream and tacrolimus 0.1% ointment and was considered for ultraviolet therapy. She was also advised on appropriate sun protection, including use of broad-based sunscreens containing zinc oxide or titanium dioxide.

Suggested Reading Crunkhorn S. Autoimmune disease: reversing vitiligo. Nat Rev. Drug Discov. 2018;17(9):622. Ezzedine K, Eleftheriadou V, Whitton M, van Geel N.  Vitiligo. Lancet. 2015;386(9988):74–84. Li CY, Dai YX, Chen YJ, Chu SY, Chen TJ, Wu CY, Chen CC, Lee DD, Chang YT.  Cancer risks in vitiligo patients: a nationwide population-based study in Taiwan. Int J Environ Res Public Health. 2018;15(9):E1847. Lotti T, D’Erme AM. Vitiligo as a systemic disease. Clin Dermatol. 2014;32(3):430–4.

Chapter 7 Case 7: Skin of Scalp, R/O Tinea vs. Seborrheic Dermatitis Dong-Lin Xie, Tam H. Nguyen, and Robert A. Norman

History and Clinical A 7-year-old African American female presented with an itchy and scaly scalp. The patient first noticed the itchiness and flaking several weeks ago. The patient’s mother also reported some areas of hair loss on the scalp at sites where the patient was scratching the most.

D.-L. Xie (*) Ruffolo, Hooper & Associates, M.D, P.A., Tampa, FL, USA e-mail: [email protected] T. H. Nguyen DO, PGY-1 Larkin Community Hospital, Miami, FL, USA R. A. Norman Department of Dermatology, Nova Southeastern University, Tampa, FL, USA Center for Geriatric Dermatology and Neuro-Dermatology, Tampa, FL, USA © Springer Nature Switzerland AG 2020 D.-L. Xie (ed.), Clinical Cases in Dermatopathology, Clinical Cases in Dermatology, https://doi.org/10.1007/978-3-030-28807-5_7

41

42

D.-L. Xie et al.

Figure 7.1  Patches of alopecia on the scalp with diffuse scale

Physical Examination The patient had non-inflammatory patches of alopecia on the scalp with diffuse scale (Fig. 7.1). Hair pull test was negative.

Clinical Figures Clinical Differential Diagnosis • • • •

Tinea Capitis Seborrheic Dermatitis Alopecia Areata Trichotillomania

Chapter 7.  Case 7: Skin of Scalp, R/O Tinea…

43

Histopathology This punch biopsy section from scalp reveals moderate mostly deep mixed granulomatous inflammation, centered on follicles (Fig. 7.2 shows a panoramic view of the entire section and Figs. 7.3, 7.4, and 7.5 are variable higher magnifications of the selected areas with inflammation of hair follicles). Fungal organisms are present in some hair shafts with arthrospores and hyphae elements. The organisms are subtle in histologic section and are obvious as can be seen in PAS stained sections (Figs. 7.6 and 7.7).

Final Diagnosis –– Tinea capitis with granulomatous inflammation (Majocchi granuloma).

Figure 7.2  Panoramic view of granulomatous inflammation under low magnification

44

D.-L. Xie et al.

Figure 7.3  Granulomatous inflammation in follicular and perifollicular areas under medium magnification

Figure 7.4  Edge of specimen with hair shaft and subtle hint of fungal infected hair shaft under medium magnification

Chapter 7.  Case 7: Skin of Scalp, R/O Tinea…

45

Figure 7.5  Same area with higher magnification showing subtle hint of fungal infected hair shaft in H&E sections

Figure 7.6  PAS stain is positive for fungi within hair shaft at one edge of the biopsy specimen

46

D.-L. Xie et al.

Figure 7.7  PAS stain is positive for fungi within two additional hair shafts in hair follicles

Case Discussion • Most cases of dermatophytosis have fungal organisms in the horny layers or in the stratum corneum and not in hair shafts. The presence of neutrophils in the stratum corneum should alert the pathologist to perform PAS or GMS to exclude fungal infection. • Even in the presence of abundant fungal organisms in the hair shafts, the organisms can be overlooked on H&E sections, because of lack of color contrast and the location (inside hair shafts), especially when there is no appropriate clinical history. Frequently after seeing a positive PAS stain, and looking back at the H&E sections, one can better appreciate the presence of the organisms on the H&E sections.

Chapter 7.  Case 7: Skin of Scalp, R/O Tinea…

47

• Some fungal organisms, T. rubrum and T. verrucosum, may cause nodular perifolliculitis, also known as Majocchi granuloma. The findings in this case may represent Majocchi granuloma.

Clinical Management Based on histopathological and clinical correlation, a diagnosis of tinea capitis was made with possible Majocchi granuloma. The patient was prescribed oral griseofulvin and topical ketoconazole shampoo for treatment.

Suggested Reading Chen X, Jiang X, Yang M, Bennett C, Gonzalez U, Lin X, Hua X, Xue S, Zhang M. Systemic antifungal therapy for tinea capitis in children: an abridged Cochrane review. J Am Acad Dermatol. 2017;76(2):368–74. Hay R.  Therapy of skin, hair and nail fungal infections. J Fungi (Basel). 2018;4(3):E99. Elghblawi E. Tinea capitis in children and trichoscopic criteria. Int J Trichology. 2017;9(2):47–9. Veasey JV, Muzy GSC.  Tinea capitis: correlation of clinical presentations to agents identified in mycological culture. An Bras Dermatol. 2018;93(3):465–6.

Chapter 8 Case 8: Skin Lesion of Abdomen, Clinically Elevated Dong-Lin Xie, Tam H. Nguyen, and Robert A. Norman

History and Clinical A 60-year-old female with an extensive past medical history of a hereditary neuropathy, congestive heart failure, type 2 diabetes mellitus, chronic obstructive pulmonary disease and schizophrenia presented with a rash on her abdomen for ­several weeks. She noted the rash was extremely itchy and flared every few months. Nothing seemed to alleviate or aggravate the itching.

D.-L. Xie (*) Ruffolo, Hooper & Associates, M.D, P.A., Tampa, FL, USA e-mail: [email protected] T. H. Nguyen DO, PGY-1 Larkin Community Hospital, Miami, FL, USA R. A. Norman Department of Dermatology, Nova Southeastern University, Tampa, FL, USA Center for Geriatric Dermatology and Neuro-Dermatology, Tampa, FL, USA © Springer Nature Switzerland AG 2020 D.-L. Xie (ed.), Clinical Cases in Dermatopathology, Clinical Cases in Dermatology, https://doi.org/10.1007/978-3-030-28807-5_8

49

50

D.-L. Xie et al.

Physical Examination The patient had a diffuse erythematous papular rash with some discrete pustules localized on the abdomen. No similar skin findings were found elsewhere on the body.

Clinical Differential Diagnosis • • • •

Bacterial Folliculitis Pityrosporum Folliculitis Allergic Contact Dermatitis Atopic Dermatitis

Histopathology Sections of the biopsy revealed a cup shaped lesion simulating a dilated follicular infundibulum containing compact ortho- and parakeratotic cornified cells/keratotic material mixed with basophilic material, inflammatory cells (mostly neutrophils), and extruding collagen and elastic fibers (Figs.  8.1 and 8.2). PAS stain was also performed and was negative for fungal organisms. Extruding, also known as transepidermal elimination of collagen and elastic fibers can be demonstrated by trichrome stain for collagen (Fig. 8.3) and Verhoeff-Van Gieson (VVG) for elastic fibers (Fig. 8.4).

Final Diagnosis –– Perforating disorder, favoring perforating folliculitis.

Case Discussion • The histologic findings are consistent with a perforating disorder, which can be seen in specific conditions and non-­

Chapter 8.  Case 8: Skin Lesion of Abdomen, Clinically…

51

Figure 8.1  Cup shaped lesion, medium magnification

Figure 8.2  Different area of cup shaped lesion, medium magnification

52

D.-L. Xie et al.

Figure 8.3 Trichrome stain, transepidermal elimination of collage fibers through epidermis

Figure 8.4 VVG stain, transepidermal elimination of elastic fiber through epidermis

Chapter 8.  Case 8: Skin Lesion of Abdomen, Clinically…

53

specific epidermal and dermal reactive and reparative reactions, including prurigo nodularis. • Some of the conditions associated with perforating disorders can be seen including followings: –– Kyrle’s disease, which has some similarity to porokeratosis of Mibelli with abnormal epidermal keratinization. –– Perforating folliculitis. –– Elastosis perforans surpiginosum (EPS). –– Reactive perforating collagenosis. –– Perforating disorder secondary to chronic renal disease or diabetes.

Clinical Management Based on histopathological and clinical correlation, a diagnosis of a perforating disorder was made. The patient was prescribed mupirocin 2% ointment, clobetasol 0.05% ointment, and cetirizine.

Suggested Reading Arora K, Hajirnis KA, Sawant S, Sabnis V, Gaikward N. Perorating disorders of the skin. Indian J Pathol Microbiol. 2013;56(4):355–8. Arora S, Malik A, Balki A. Reactive perforating collagenosis. Indian Dermatol Online J. 2016;7(2):139–40. Bekkali N, Gil Bistes D, Joujoux JM, Meunier L, Stoebner PE.  Acquired reactive perforating collagenosis. Ann Dermatol Venereol. 2014;141(2):170–2. Kim RH, Kwa M, Adams S, Meehan SA, Stein JA. Giant acquired reactive perforating collagenosis in a patient with diabetes mellitus and metastatic breast carcinoma. JAAD Case Rep. 2016;2(1):22–4.

Chapter 9 Case 9: Skin of Right Inferior Eyelid Lesion, R/O Seborrheic Keratosis Dong-Lin Xie, Tam H. Nguyen, and Robert A. Norman

History and Clinical A 59-year-old African American female presented for evaluation of a growth on her right lower eyelid. The growth was present for many years. She denied any pain, pruritus, or recent changes of the lesion.

Physical Examination The patient had a solitary, smooth, slightly darker skin colored papule inferior to the lower eyelid (Fig. 9.1).

D.-L. Xie (*) Ruffolo, Hooper & Associates, M.D, P.A., Tampa, FL, USA e-mail: [email protected] T. H. Nguyen DO, PGY-1 Larkin Community Hospital, Miami, FL, USA R. A. Norman Department of Dermatology, Nova Southeastern University, Tampa, FL, USA Center for Geriatric Dermatology and Neuro-Dermatology, Tampa, FL, USA © Springer Nature Switzerland AG 2020 D.-L. Xie (ed.), Clinical Cases in Dermatopathology, Clinical Cases in Dermatology, https://doi.org/10.1007/978-3-030-28807-5_9

55

56

D.-L. Xie et al.

Figure 9.1  Solitary, smooth, papule inferior to the lower eyelid

Clinical Figure(s) Clinical Differential Diagnosis • • • •

Fibroma Neurofibroma Xanthoma Dermal Nevus

Histopathology Sections of this shave skin biopsy revealed a dome shaped cellular infiltrate that filled the entirely dermal space (Fig. 9.2). Higher magnifications showed a mixed epithelioid and plumped spindled cells with occasional multinucleated cells (Fig.  9.3). Some of the multinucleated cells demonstrated foamy cytoplasm and a ‘wreath like’ array of the nuclei, i.e., Touton giant cells (Figs. 9.4 and 9.5).

Chapter 9.  Case 9: Skin of Right Inferior Eyelid Lesion…

57

Figure 9.2  Panoramic view of the entire biopsy, low magnification

Figure 9.3  The lesion is very cellular, medium magnification

58

D.-L. Xie et al.

Figure 9.4  Giant cells with foamy cytoplasm, medium high magnification

Figure 9.5  Touton giant cells with ‘wreath like’ array of the nuclei, high magnification

Chapter 9.  Case 9: Skin of Right Inferior Eyelid Lesion…

59

Final Diagnosis –– Xanthogranuloma.

Case Discussion • Xanthogranuloma is a similar lesion to juvenile xanthogranuloma (JXG) in children. • Solitary juvenile xanthogranuloma may not have clinical significance. However, multiple juvenile xanthogranulomas have been associated with underlying conditions in some patients. • Eruptive xanthoma is a histologic differential consideration. Hyperlipidemia should be clinically excluded. • Though this is located at the lower eyelid, it is not a xanthelasma.

Clinical Management Based on histopathological and clinical correlation, a diagnosis of a xanthogranuloma was made. Because eruptive xanthoma is a histologic differential consideration, labs were ordered to assess lipid levels. Notably, hypertriglyceridemia is a common cause of eruptive xanthomas.

Suggested Reading Frings VG, Goebeler M, Wobser M. Dermpath & clinic: juvenile xanthogranuloma. Eur J Dermatol. 2018;28(3):429–31. Ladha MA, Haber RM.  Giant juvenile xanthogranuloma: case report, literature review, and algorithm for classification. J Cutan Med Surg. 2018;22(5):488–94. Santiago L, Pinho A, Cardoso JC.  Eruptive xanthomas: a cardinal manifestation of serious metabolic disease. Acta Med Port. 2018;31(4):219–22. Stark M, Sutart J. Eruptive xanthoma in the setting of hypertriglyceridemia and pancreatitis. Am J Emerg Med. 2018;36(8):1524.

Chapter 10 Case 10: Four-Year-Old Female with Left Lateral Trunk Lesions Dong-Lin Xie, Tam H. Nguyen, and Robert A. Norman

History and Clinical A 4-year-old female with no significant past medical history presented with dark raised lesions on the stomach and back since 2–3 months of age. The lesions would come and go and were not itchy. The patient had never had them evaluated before. She met all age appropriate milestones, and there was no personal or family history of allergies. Review of systems was negative for any constitutional symptoms, diarrhea, or wheezing.

D.-L. Xie (*) Ruffolo, Hooper & Associates, M.D, P.A., Tampa, FL, USA e-mail: [email protected] T. H. Nguyen DO, PGY-1 Larkin Community Hospital, Miami, FL, USA R. A. Norman Department of Dermatology, Nova Southeastern University, Tampa, FL, USA Center for Geriatric Dermatology and Neuro-Dermatology, Tampa, FL, USA © Springer Nature Switzerland AG 2020 D.-L. Xie (ed.), Clinical Cases in Dermatopathology, Clinical Cases in Dermatology, https://doi.org/10.1007/978-3-030-28807-5_10

61

62

D.-L. Xie et al.

Physical Examination The patient had red to brown, hyperpigmented macules distributed on the trunk and neck (Fig.  10.1). When the skin lesions were stroked, they became more erythematous and slightly edematous, signifying a positive Darier’s sign (Fig. 10.2).

Clinical Figures

Figure 10.1  Red to brown, hyperpigmented macules on the trunk

Chapter 10.  Case 10: Four-Year-Old Female with Left…

63

Figure 10.2  Positive Darier’s sign upon stoke

Clinical Differential Diagnosis • • • •

Dermatographism Urticaria Pigmentosa Urticarial vasculitis Erythema Multiforme

Histopathology Superficial and mid dermal interstitial dermal infiltrate with pale pink to light gray cells are present (Figs. 10.3 and 10.4), mimicking inflammatory cells and nevoid cells. Under higher magnification, the cells have round nuclei and faint granular cytoplasm (Fig. 10.5). Giemsa (toluidine blue, Fig. 10.6) demonstrates metachromatic granules purple granules in the cytoplasm. Immunostain is positive for CD-117.

64

D.-L. Xie et al.

Figure 10.3  Superficial and mid dermal interstitial dermal infiltrate, mimicking inflammation and nevoid cells, low magnification

Figure 10.4  Superficial and mid dermal interstitial dermal infiltrate, mimicking inflammation and nevoid cells, medium magnification

Chapter 10.  Case 10: Four-Year-Old Female with Left…

65

Figure 10.5  Cells with faint purple granular cytoplasm, high magnification

Figure 10.6 Toluidine blue stain showing metachromatic purple granules

66

D.-L. Xie et al.

Final Diagnosis –– Urticaria pigmentosa.

Case Discussion • Urticaria pigmentosa is a mast cell proliferative disease. Most of the cases, lesions arising in infancy and early childhood are localized disease and often improve or clear (spontaneous regression) at the puberty. • Urticaria pigmentosa arising during adolescence or adult life is usually a systemic disease, but most of them are rather static in their course and do not spontaneously regress. A few of those patients will have progression in the form of infiltrating internal organs. Real mast cell leukemia is rare. • A macular type, telangiectasia macularis eruptiva perstans (TMEP) usually limits to upper dermis in the perivascular area and present in adults. • Without appropriate clinical history, urticaria pigmentosa can be mistaken as nevus or non-specific dermatitis under low magnification. The macular type, telangiectasia macularis eruptiva perstans (TMEP) can be easily overlooked when the sections are read at low magnification. Giemsa (toluidine blue) and CD117 (c-kit) are helpful, especially in this type.

Clinical Management Based on histopathological and clinical correlation, a diagnosis of urticaria pigmentosa was made. The patient was prescribed cetirizine, topical tacrolimus 0.03%, and recommended to use emollients and Sarna lotion. Follow up eight weeks later revealed noticeable improvement in induration and pigmentation of the lesions.

Chapter 10.  Case 10: Four-Year-Old Female with Left…

67

Suggested Reading Güneş Bilgili S, Karadağ AS, Takci Z, Çalka Ö, Kösem M. Comparison of cutaneous mastocytosis with onset in children and adults. Turk J Med Sci. 2014;44(3):504–10. Heinze A, Kuemmet TJ, Chiu YE, Galbraith SS.  Longitudinal study of pediatric urticaria pigmentosa. Pediatr Dermatol. 2017;34(2):144–9. Mashiah J, Harel A, Bodemer C, Hadj-Rabia S, Goldberg I, Sprecher E, Kutz A. Topical pimecrolimus for pediatric cutaneous mastocytosis. Clin Exp Dermatol. 2018;43(5):559–65.

Chapter 11 Case 11: Skin Lesion Left Upper Arm, R/O Cyst Dong-Lin Xie, Tam H. Nguyen, and Robert A. Norman

History and Clinical A 17-year-old male presented with concerns of a mass on his left arm for 2  years. The patient noticed that the mass had been enlarging in the past several months. He denied any pain, pruritus, or discharge. He had no previous history of similar lesions.

Physical Examination The patient had a 1.5 cm grey, smooth, dome shaped nodule with a central punctum on the brachial region of his left upper arm (Fig.  11.1). There was no evidence of tenderness upon palpation, or signs of infection.

D.-L. Xie (*) Ruffolo, Hooper & Associates, M.D, P.A., Tampa, FL, USA e-mail: [email protected] T. H. Nguyen DO, PGY-1 Larkin Community Hospital, Miami, FL, USA R. A. Norman Department of Dermatology, Nova Southeastern University, Tampa, FL, USA Center for Geriatric Dermatology and Neuro-Dermatology, Tampa, FL, USA © Springer Nature Switzerland AG 2020 D.-L. Xie (ed.), Clinical Cases in Dermatopathology, Clinical Cases in Dermatology, https://doi.org/10.1007/978-3-030-28807-5_11

69

70

D.-L. Xie et al.

Clinical Figure(s)

Figure 11.1  Smooth, dome shaped nodule with a central punctum on the left upper arm

Clinical Differential Diagnosis • • • •

Epidermal Inclusion Cyst Lipoma Neurofibroma Dermatofibroma

Histopathology A proliferation of uncircumscribed groups and cords of cells with round nuclei in most of cells and abundant pink cytoplasm distributed between collagen bundles (Figs. 11.2 and 11.3). The cytoplasm has pink granular material with occasional larger pink granular droplets in the cytoplasm (Fig.  11.4). No overt mitoses or significant spindling of the

Chapter 11.  Case 11: Skin Lesion Left Upper Arm…

71

Figure 11.2  Superficial portion of the tumor cells with abundant pink cytoplasm, distributed between collagen bundles, low magnification

Figure 11.3  Deep portion of the tumor, not encapsulated, low magnification

72

D.-L. Xie et al.

Figure 11.4  Granular cytoplasm and larger pink droplets, high magnification

tumor cells are seen. Immunostain for S-100 is positive in the tumor cells (Fig.  11.5). PAS with diastase is positive in the cytoplasm granules in the tumor cells (Fig. 11.6).

Final Diagnosis –– Granular cell tumor.

Case Discussion • In granular cell tumor, the overlying epidermis may have pseudoepitheliomatous hyperplasia and can be mistaken as well differentiated squamous cell carcinoma. If the biopsy is superficial, the true underlying tumor, which cause pseudoepitheliomatous hyperplasia, can be overlooked.

Chapter 11.  Case 11: Skin Lesion Left Upper Arm…

Figure 11.5  S-100 is positive for nuclei and granular cytoplasm

Figure 11.6  PAS with diastase is positive in the granules

73

74

D.-L. Xie et al.

• The cells of granular cell tumor are considered neural crest origin, hence S-100 positive. The granules are positive for PAS with diastase stain and are lysosomes. • Granular cell tumor is benign, but unencapsulated and uncircumscribed. Complete removal of the residual lesion is the treatment of cure.

Clinical Management Based on histopathological and clinical correlation, a diagnosis a granular cell tumor was made. The tumor was removed with a wide excision.

Suggested Reading Hong SB, Yang MH, Lee MH, Haw CR. Dermatofibroma-like atypical granular cell tumor. Acta Derm Venereol. 2005;85(2):179–80. Lee MW, Change SE, Song KY, Choi JH, Sung KJ, Moon KC, Koh JK. S-100-negative atypical granular cell tumor: report of a case. Int J Dermatol. 2002;41(3):168–70.

Chapter 12 Case 12: Skin of Back, R/O Cancer vs. Dermatitis, vs. Tinea Dong-Lin Xie, Tam H. Nguyen, and Robert A. Norman

History and Clinical An 88-year-old female with a history of left breast cancer presented with a rash on the back and abdomen for several months. She was initially diagnosed with breast cancer 22  years ago and was treated with a mastectomy and ­endocrine adjuvant therapy. The rash was not painful, or pruritic, however, it did not seem to get better since its onset several months ago.

D.-L. Xie (*) Ruffolo, Hooper & Associates, M.D, P.A., Tampa, FL, USA e-mail: [email protected] T. H. Nguyen DO, PGY-1 Larkin Community Hospital, Miami, FL, USA R. A. Norman Department of Dermatology, Nova Southeastern University, Tampa, FL, USA Center for Geriatric Dermatology and Neuro-Dermatology, Tampa, FL, USA © Springer Nature Switzerland AG 2020 D.-L. Xie (ed.), Clinical Cases in Dermatopathology, Clinical Cases in Dermatology, https://doi.org/10.1007/978-3-030-28807-5_12

75

76

D.-L. Xie et al.

Physical Examination The patient had multiple large, poorly demarcated, indurated, erythematous plaques involving large areas of her mid back, especially on the left side (Fig.  12.1). Similar findings were also noted diffusely on the abdomen (Fig. 12.2). Figure 12.1  Poorly demarcated, indurated, erythematous plaques in the left back

Figure 12.2  Poorly demarcated, indurated, erythematous plaques in the left abdomen

Chapter 12.  Case 12: Skin of Back, R/O Cancer…

77

Clinical Figures

Figure 12.3  Slightly increased interstitial cells, low magnification

Clinical Differential Diagnosis • • • •

Cutaneous metastasis Dermatofibrosarcoma Protuberans Neurofibrosarcoma Angiosarcoma

Histology and Diagnosis Histopathology This superficial shave skin biopsy reveals slightly increased interstitial and perivascular cells and is overall rather insignificant (Fig. 12.3). Under medium high magnification, some cells appear to have hyperchromatic nuclei (Fig.  12.4). Immuno histochemical (IHC) studies reveal that the cells are negative for CK5/6 (Fig.  12.5) and Mart-1 (Fig.  12.6) and positive for CK7 (Fig. 12.7), estrogen receptor (ER) (Fig. 12.8) and occasional cells positive for progesterone receptor (PR) (Fig. 12.9). Occasional cells have hyperchromatic nuclei.

78

D.-L. Xie et al.

Figure 12.4  Slightly increased interstitial cells, medium high magnification

Figure 12.5  Immuno histochemical stain for CK5/6, negative in the suspicious cells

Chapter 12.  Case 12: Skin of Back, R/O Cancer…

79

Figure 12.6  Immuno histochemical stain for Mart-1, negative in the suspicious cells

Figure 12.7 Immuno histochemical stain for CK7, positive in the suspicious cells

80

D.-L. Xie et al.

Figure 12.8  Immuno histochemical stain for ER (estrogen receptor), positive nuclei in the suspicious cells

Figure 12.9 Immuno histochemical stain for PR (progesterone receptor), weakly positive nuclei in some of the suspicious cells

Chapter 12.  Case 12: Skin of Back, R/O Cancer…

81

Final Diagnosis –– Metastatic breast carcinoma.

Case Discussion • Metastatic breast carcinoma and other metastatic adenocarcinoma of skin can be subtle and overlooked. Frequently, the biopsy specimen is received to R/O dermatitis. Careful examination of the sections is critical especially when the biopsy is superficial. • Immuno studies to evaluate possible primary are usually performed. Selection of immunostains are based on sites of the specimens, morphology and gender. • In this biopsy, the tumor cells are subtle in the histologic sections and does not have structural resemblance of breast carcinoma. However, immuno studies provide evidence of breast origin, and also confirmed with clinical history that the patient had diagnosis of breast cancer more than 20 years ago. • In other cases when the histologic features and the immunologic findings are consistent with adenocarcinoma but cannot decide the origin or cannot differentiate primary skin vs. metastatic adenocarcinoma, the combination of CK5/6 and CK7 stains may provide some suggestions: primary skin adnexal adenocarcinomas often express strong positivity for CK5/6 and focal positivity for CK7, while metastatic adenocarcinomas are strongly positive for CK7 and CK5/6 is positive in minority of metastatic adenocarcinomas.

Clinical Management Based on histopathological and clinical correlation, a diagnosis of cutaneous metastatic breast cancer was made. The patient was referred to a cancer center for further evaluation and treatment.

82

D.-L. Xie et al.

Suggested Reading Chisti MA, Alfadley AA, Banka N, Ezzat A.  Cutaneous metastasis from breast carcinoma: a brief report of a rare variant and proposed morphological classification. Gulf J Oncolog. 2013;1(14):90–4. Ferreira VA, Spelta K, Diniz LM, Lucas EA.  Exuberant case of cutaneous metastasis of breast cancer. An Bras Dermatol. 2018;93(3):429–31. Madabhavi I, Kadakol N, Chavan C, Sarkar M. Multiple subcutaneous nodules leading to diagnosis of colon cancer. Middle East J Dig Dis. 2018;10(3):188–91.

Chapter 13 Case 13: Skin of Right Wrist, R/O Foreign Object Dong-Lin Xie, Tam H. Nguyen, and Robert A. Norman

History and Clinical An 80-year-old male presented with a tender nodule on his right wrist. He stated that one week ago, he was gardening when he was struck by a thorn in that location. Since that time, the nodule grew slowly every day. He denied any ­discharge or pruritis in the lesion. The patient did not experience any constitutional symptoms like fever or malaise. No other similar lesions were reported elsewhere on the body.

D.-L. Xie (*) Ruffolo, Hooper & Associates, M.D, P.A., Tampa, FL, USA e-mail: [email protected] T. H. Nguyen DO, PGY-1 Larkin Community Hospital, Miami, FL, USA R. A. Norman Department of Dermatology, Nova Southeastern University, Tampa, FL, USA Center for Geriatric Dermatology and Neuro-Dermatology, Tampa, FL, USA © Springer Nature Switzerland AG 2020 D.-L. Xie (ed.), Clinical Cases in Dermatopathology, Clinical Cases in Dermatology, https://doi.org/10.1007/978-3-030-28807-5_13

83

84

D.-L. Xie et al.

Physical Examination The patient had a solitary, poorly defined, mildly erythematous dermal nodule on the right radial region of his wrist (Fig.  13.1). Palpation of the mass elicited pain. No visible punctum was seen.

Clinical Figure(s)

Figure 13.1  Poorly defined, mildly erythematous dermal nodule on the right wrist

Chapter 13.  Case 13: Skin of Right Wrist, R/O Foreign…

85

Clinical Differential Diagnosis • • • •

Foreign body reaction Ganglion cyst Lipoma Epidermal Inclusion Cyst

Histopathology Sections from this punch biopsy of skin reveal suppurative granulomatous infiltrate in deep dermis with neutrophils, lymphocytes, plasma cells, histiocytes and multinucleated cells surrounding a foreign appearing material (Figs. 13.2 and 13.3). The material is polarizable (Fig. 13.4). PAS stain is positive for fungal hyphae. Some of the hyphae have brown color (Fig. 13.5).

Figure 13.2  Suppurative granuloma with foreign material, low magnification

86

D.-L. Xie et al.

Figure 13.3  Acute inflammatory cells lining edges of foreign body with vaguely visible organisms in H&E section, high magnification

Figure 13.4 The foreign material is polarizable under polarizing microscope

Chapter 13.  Case 13: Skin of Right Wrist, R/O Foreign…

87

Figure 13.5  PAS positive for fungi and some brown colored fungi

Final Diagnosis –– Foreign body suppurative granulomatous inflammation with phaeohyphomycosis.

Case Discussion • Some people define phaeohyphomycosis more strictly as deep subcutaneous or systemic infection with pigmented fungal organism. Sometimes, phaeohyphomycosis is loosely defined as pigmented fungal organisms in the tissue, including non-pathogenic organisms, such as ­ alternariosis. • The polarizable foreign material has morphologic features of plant material, such as wood/thorn. • The organisms in this phaeohyphomycosis are probably non-pathogenic environmental organisms and were introduced into the skin through traumatic implantation of

88

D.-L. Xie et al.

wood/thorn, probably through gardening. The organisms may colonize in the dermis. The process is usually localized.

Clinical Management Based on histopathological and clinical correlation, a diagnosis of a foreign body reaction to fungal organisms was made. The patient reported significant improvement in pain and inflammation since the biopsy (which removed most of the lesion) and was advised to follow up as needed.

Suggested Reading Behera B,Thomas E, Kumari R,Thappa DM, Jinkala S. Polymorphous presentation of subcutaneous phaeohyphomycosis: a rare occurrence. Int J Dermatol. 2018;57(2):e1–3. Jinkala SR, Basu D, Neelaiah S, Stephen N, Bheemanati Hanuman S, Singh R.  Subcutaneous phaeohyphomycosis: a clinical mimic of skin and soft tissue neoplasms—A descriptive study from India. World J Surg. 2018;42:3861–6. https://doi.org/10.1007/ s00268-018-4745-0.

Chapter 14 Case 14: Left Parietal Skin, R/O Dysplastic Nevus Dong-Lin Xie, Tam H. Nguyen, and Robert A. Norman

History and Clinical A 32-year-old female presented for evaluation and removal of a mole on her left scalp. She stated that the mole was present since early childhood, and did not undergo any recent change or cause pain or pruritus. There was no personal history of any skin cancer.

D.-L. Xie (*) Ruffolo, Hooper & Associates, M.D, P.A., Tampa, FL, USA e-mail: [email protected] T. H. Nguyen DO, PGY-1 Larkin Community Hospital, Miami, FL, USA R. A. Norman Department of Dermatology, Nova Southeastern University, Tampa, FL, USA Center for Geriatric Dermatology and Neuro-Dermatology, Tampa, FL, USA © Springer Nature Switzerland AG 2020 D.-L. Xie (ed.), Clinical Cases in Dermatopathology, Clinical Cases in Dermatology, https://doi.org/10.1007/978-3-030-28807-5_14

89

90

D.-L. Xie et al.

Figure 14.1  Irregular, skin colored nodule on the left midline parietal scalp

Physical Examination The patient had an irregular, skin colored nodule on the left midline parietal scalp (Fig. 14.1).

Clinical Figure(s) Clinical Differential Diagnosis • • • •

Nevus sebaceous Melanocytic nevus Xanthogranuloma Seborrheic keratosis

Chapter 14.  Case 14: Left Parietal Skin, R/O Dysplastic…

91

Histopathology Sections from this shave biopsy of skin reveal a dome shaped lesion with fibrotic dermal tissue, adipose cells/adipocytes (Fig. 14.2), disorganized follicular sebaceous elements (Figs. 14.3 and 14.4), melanocytic cells extending between collagen bundles and around adnexal structure with congenital features (Fig. 14.5), and loose spindled mesenchymal cells with myxoid stroma (Fig. 14.6). A panoramic view of most of the elements of the lesion is demonstrated in Fig. 14.7.

Figure 14.2  Adipocytes in superficial dermis, low magnification

92

D.-L. Xie et al.

Figure 14.3  Disorganized follicular sebaceous elements, low magnification

Figure 14.4  Another area of disorganized follicular sebaceous elements, low magnification

Chapter 14.  Case 14: Left Parietal Skin, R/O Dysplastic…

93

Figure 14.5  Melanocytic cells extending between collagen bundles and around adnexal structure, medium magnification

Figure 14.6  Mesenchymal cells with scattered spindle cells myxoid stroma, medium high magnification

94

D.-L. Xie et al.

Figure 14.7  Panoramic view of most of the elements in this lesion, low magnification

Final Diagnosis –– Combined congenital melanocytic nevus and mesenchymal hamartoma.

Case Discussion • This is a benign lesion with combined congenital nevus and ectopic tissue/hamartoma. • The ectopic tissue/hamartoma is mostly composed of mesenchymal remnants. Usually the lesion is long standing without significant changes over years, commonly seen on the head and face and may be irritating. • Combined congenital nevus with nevus lipomatosus superficial is a histologic differential consideration, also a benign lesion.

Chapter 14.  Case 14: Left Parietal Skin, R/O Dysplastic…

95

Clinical Management Based on histopathological and clinical correlation, a diagnosis of a nevus with an associated hamartoma/trichodiscoma was made. The patient was reassured about the benign nature of the tumor without additional treatment.

Suggested Reading Mertens J, Schubert C.  Small papular tumors of the perifollicular and perivascular connective tissue of the head and neck area. HNO. 1991;39(7):266–70. Misago N, Kimura T, Narisawa Y.  Fibrofolliculoma/trichodiscoma and fibrous papule (perifollicular fibroma/angiofibroma): a revaluation of the histopathological and immunohistochemical features. J Cutan Pathol. 2009;36(9):943–51. Tellechea O, Cardoso JC, Reis JP, Ramos L, Gameiro AR, Coutinho I, Baptista AP.  Benign follicular tumors. An Bras Dermatol. 2015;90(6):780–96.

Chapter 15 Case 15: Skin of Mid Lateral Back, R/O Fibroma Dong-Lin Xie, Tam H. Nguyen, and Robert A. Norman

History and Clinical An 18-year-old male with no significant past medical history presented with a bump on his left back. The patient recalled it had been present for several years and had not enlarged or changed in any way. Recently, the bump would become irritated when he would put on or take off his shirt.

D.-L. Xie (*) Ruffolo, Hooper & Associates, M.D, P.A., Tampa, FL, USA e-mail: [email protected] T. H. Nguyen DO, PGY-1 Larkin Community Hospital, Miami, FL, USA R. A. Norman Department of Dermatology, Nova Southeastern University, Tampa, FL, USA Center for Geriatric Dermatology and Neuro-Dermatology, Tampa, FL, USA © Springer Nature Switzerland AG 2020 D.-L. Xie (ed.), Clinical Cases in Dermatopathology, Clinical Cases in Dermatology, https://doi.org/10.1007/978-3-030-28807-5_15

97

98

D.-L. Xie et al.

Figure 15.1  Solitary, mildly erythematous papule on the midline back

Physical Examination The patient had a solitary, mildly erythematous papule on the midline back. No overlying scale or erosion was seen (Fig. 15.1).

Clinical Figure(s) Clinical Differential Diagnosis • • • •

Neurofibroma Acrochordon Dermal nevus Dermatofibroma

Chapter 15.  Case 15: Skin of Mid Lateral Back, R/O…

99

Figure 15.2  Central follicle surrounded by radiating secondary follicles, low magnification

Histopathology Sections from this shave biopsy of skin reveal a dilated partially keratin-filled central hair follicle surrounded by multiple secondary follicles radiating from the central follicle (Figs.  15.2 and 15.3). The secondary hair follicles are somewhat immature with fibrotic stroma resembling fibrous root sheath (Figs. 15.4 and 15.5).

Final Diagnosis –– Trichofolliculoma.

100

D.-L. Xie et al.

Figure 15.3  Central follicle surrounded by radiating secondary follicles, low medium magnification

Figure 15.4  Immature secondary follicles, medium magnification

Chapter 15.  Case 15: Skin of Mid Lateral Back, R/O…

101

Figure 15.5 Immature radiating secondary follicles and fibrotic stroma resembling fibrous root sheath, medium magnification

Case Discussion • Trichofolliculoma is a benign tumor that usually presents on head, neck and face in adults. Some of the immature follicles have clear cells, resembling outer root sheath of a normal follicle with glycogen in the cytoplasm. • The lesion in this case is a small early form lesion. Most trichofolliculomas are larger than the lesion biopsied in this case.

Clinical Management Based on histopathological and clinical correlation, a diagnosis of a trichofolliculoma was made. The patient was reassured about the benign nature of the tumor without additional treatment.

102

D.-L. Xie et al.

Suggested Reading Garcia-Garcia SC, Villarreal-Martinez A, Guerrero-Gonzalez GA, Miranda-Maldonado I, Ocampo-Candiani J.  Dermoscopy of trichofolliculoma: a rare hair follicle hamartoma. J Eur Acad Dermatol Venereol. 2017;31(2):e123–4. Romero-Perez D, Garcia-Bustinduy M, Cribier B. Clinicopathologic study of 90 cases of trichofolliculoma. J Eur Acad Dermatol Venereol. 2017;31(3):e141–2. Tellechea O, Cardoso JC, Reis JP, Ramos L, Gameiro AR, Coutinho I, Baptista AP.  Benign follicular tumors. An Bras Dermatol. 2015;90(6):780–96.

Chapter 16 Case 16: Skin of Upper Abdomen, R/O Squamous Cell Carcinoma vs. Basal Cell Carcinoma Dong-Lin Xie, Tam H. Nguyen, and Robert A. Norman

History and Clinical An 84-year-old female presented with a red lesion on her abdomen for several years. The lesion was not itchy, or painful. However, it did grow in recent months and did not go away with topical steroids. No drainage was reported. The patient had no previous history of skin cancer.

D.-L. Xie (*) Ruffolo, Hooper & Associates, M.D, P.A., Tampa, FL, USA e-mail: [email protected] T. H. Nguyen DO, PGY-1 Larkin Community Hospital, Miami, FL, USA R. A. Norman Department of Dermatology, Nova Southeastern University, Tampa, FL, USA Center for Geriatric Dermatology and Neuro-Dermatology, Tampa, FL, USA © Springer Nature Switzerland AG 2020 D.-L. Xie (ed.), Clinical Cases in Dermatopathology, Clinical Cases in Dermatology, https://doi.org/10.1007/978-3-030-28807-5_16

103

104

D.-L. Xie et al.

Physical Examination The patient had a discrete, well-demarcated, 2.5 cm by 2.7 cm red to pink, irregular verrucous plaque on the upper abdomen. There were varying levels of colors within the plaque, with brownish hues on the right border and right inferior portions of the lesion (Fig. 16.1).

Clinical Figure(s) Clinical Differential Diagnosis • Basal cell carcinoma • Squamous cell carcinoma

Figure 16.1  Discrete, well-demarcated red to pink to brown, irregular verrucoid plaque on the upper abdomen

Chapter 16.  Case 16: Skin of Upper Abdomen, R/O…

105

• Poroma • Dermatofibroma

Histopathology Sections of this shave biopsy reveal a demarcated epidermal proliferation composed of clonal squamoid cells with occasional small eccrine ducts identified (Figs.  16.2 and 16.3). Underlying higher magnification, More sharp clonal appearance and high nuclear to cytoplasm ratio are seen (high N:C ratio, Fig.  16.4). Immunohistochemical studies reveal that the cells are positive for CK5/6, negative for D2–40 and partially positive for p16 (Fig. 16.5).

Final Diagnosis –– Dysplastic/atypical intraepidermal poroma.

Figure 16.2  Vague clonal proliferation, low medium magnification

106

D.-L. Xie et al.

Figure 16.3  More demarcated clonal proliferation, medium magnification

Figure 16.4 High nuclear to cytoplasm ratio and hyperchromatic nuclei and distinctive intraepidermal clonal pattern, high magnification

Chapter 16.  Case 16: Skin of Upper Abdomen, R/O…

107

Figure 16.5  Immunohistochemical stain for p16 is positive in clonal tumor cells

Case Discussion • Intraepidermal poroma, also known as hidroacanthoma simplex is probably arising from intraepidermal acrosyringeal ducts. • This biopsy shows atypical cytology and p16 demonstrates positive stains in clonal tumor proliferation. • The histologic pattern is loosely classified into Borst-­ Jadassohn epithelioma which include hidroacanthoma simplex (intraepidermal poroma), clonal seborrheic keratosis and pagetoid squamous cell carcinoma in situ (Bowenoid). • In the presence of dysplastic cytologic features and positive p16, the differential considerations include atypical intraepidermal poroma, porocarcinoma in situ and Bowenoid squamous cell carcinoma in situ.

108

D.-L. Xie et al.

Clinical Management Based on histopathological and clinical correlation, a diagnosis of a porocarcinoma in situ was clinically considered. The patient was scheduled for a complete removal of the tumor to evaluate for clear margins.

Suggested Reading Erfut-Berge C, Erdmann M, Brauner K, Bauerschmitz J.  Eccrine porocarcinoma mimicking a venous leg ulcer. Int Wound J. 2016;13(5):1063–4. Raheemullah A, Allamaneni S, Weber S, Singh R.  Eccrine porocarcinoma presenting as a hand cyst. J Hand Surg Am. 2016;41(11):e425–7. Su P, Heng JK, Chen Wee Aw D, Tan KB. Characteristics of eccrine tumors in a tertiary institution: a 5  year retrospective study. Skinmed. 2016;14(3):175–80. Tolkachjov SN, Hocker TL, Camilleri MJ, Baum CL. Treatment of porocarcinoma with Mohs micrographic surgery: the mayo clinic experience. Dermatol Surg. 2016;42(6):745–50.

Chapter 17 Case 17: Skin of Upper Paraspinal Lesion, R/O Miliaria Rubra vs. Atopic Dermatitis Dong-Lin Xie, Tam H. Nguyen, and Robert A. Norman

History and Clinical A 57-year-old man presented for evaluation of itchy rashes that would come and go for many years. The patient stated that the rash would flare the most when he was working outside in the heat. He was previously prescribed an antihistamine and clotrimazole by his primary care provider, which transiently relieved his symptoms.

D.-L. Xie (*) Ruffolo, Hooper & Associates, M.D, P.A., Tampa, FL, USA e-mail: [email protected] T. H. Nguyen DO, PGY-1 Larkin Community Hospital, Miami, FL, USA R. A. Norman Department of Dermatology, Nova Southeastern University, Tampa, FL, USA Center for Geriatric Dermatology and Neuro-Dermatology, Tampa, FL, USA © Springer Nature Switzerland AG 2020 D.-L. Xie (ed.), Clinical Cases in Dermatopathology, Clinical Cases in Dermatology, https://doi.org/10.1007/978-3-030-28807-5_17

109

110

D.-L. Xie et al.

Physical Examination The patient had erythematous papules and some discrete pustules that coalesced into plaques at the mid scapular region.

Clinical Differential Diagnosis • • • •

Miliaria Atopic Dermatitis Folliculitis Herpes simplex

Histopathology Sections of this shave biopsy of skin reveal inflammatory infiltrate centered on a follicle and acrosyringeal duct (Figs. 17.1 and 17.2). A collection of neutrophils is present in

Figure 17.1  Inflammation centered on a follicle and acrosyringium, low magnification

Chapter 17.  Case 17: Skin of Upper Paraspinal Lesion…

111

Figure 17.2  Inflammation centered on a follicle and acrosyringium, deeper level section better showing follicle and acrosyringium, low medium magnification

the subcorneal area at the opening of an acrosyringium with eccrine duct spongiosis (Fig.  17.3). A mixed inflammatory infiltrate is more prominent surrounding acrosyringeal duct with neutrophils invading into the ductal epithelium (Fig. 17.4). PAS stain is negative for fungal organisms.

Final Diagnosis –– Consistent with miliaria pustulosa.

Case Discussion • Miliaria is a group of disease caused by obstruction of sweat duct. Three types of miliaria can be seen histologically.

112

D.-L. Xie et al.

Figure 17.3  Neutrophils are present at the ostium of a follicle and acrosyringium, medium magnification

Figure 17.4  Neutrophils in the ductal epithelium, high magnification

Chapter 17.  Case 17: Skin of Upper Paraspinal Lesion…

113

• Miliaria alba/miliaria crystallina: most superficial obstruction at the stratum corneum, forming intracorneal small vesicle (empty space) with keratin on both roof and floor, usually cannot be diagnosed histologically. • Miliaria rubra (prickly heat): variable spongiosis or spongiotic vesiculation of epidermal acrosyringeal duct, generally difficult to diagnose on the histologic ground. • Miliaria pustulosa: neutrophils beneath stratum corneum or into sweat duct epithelium are seen. This type can be diagnosed histologically when great attention paid to the details. • In general, miliaria is an under diagnosed condition, since the changes are subtle and is frequently diagnosed as eczematous dermatitis.

Clinical Management Based on histopathological and clinical correlation, a diagnosis of miliaria pustulosa is made. The patient was prescribed an antihistamine, topical steroids, and advised to avoid exacerbating factors such as heat, humidity, and excessive sweating.

Suggested Reading Haque MS, Hailu T, Pritchett E, Cusack CA, Allen HB. The oldest new finding in atopic dermatitis: subclinical miliaria as an origin. JAMA Dermatol. 2013;149(4):436–8. Nagai H, Nishigori C.  Neutrophilic superficial eccrine ductitis: proposal of a new disease concept. Pediatr Dermatol. 2018;35(2):e105–9. Shuster S. Duct disruption, a new explanation of miliaria. Acta Derm Venereol. 1997;77(1):1–3.

Chapter 18 Case 18: Skin Right Forearm, R/O Rhus Derm Dong-Lin Xie, Tam H. Nguyen, and Robert A. Norman

History and Clinical A 20-year-old female presented for evaluation of a rash on her right arm. It began one week prior to her visit after she went hiking at a local park. Since then, the patient reported that it was spreading outwards. She recently went to the emergency room, where they prescribed her a course of oral prednisone and antibiotics, which did not improve her symptoms.

D.-L. Xie (*) Ruffolo, Hooper & Associates, M.D, P.A., Tampa, FL, USA e-mail: [email protected] T. H. Nguyen DO, PGY-1 Larkin Community Hospital, Miami, FL, USA R. A. Norman Department of Dermatology, Nova Southeastern University, Tampa, FL, USA Center for Geriatric Dermatology and Neuro-Dermatology, Tampa, FL, USA © Springer Nature Switzerland AG 2020 D.-L. Xie (ed.), Clinical Cases in Dermatopathology, Clinical Cases in Dermatology, https://doi.org/10.1007/978-3-030-28807-5_18

115

116

D.-L. Xie et al.

Physical Examination The patient had a well demarcated, linear, erythematous plaque with areas of vesiculation on her right arm (Fig. 18.1).

Figure 18.1  Demarcated, linear, erythematous plaque with areas of vesiculation on the right arm

Chapter 18.  Case 18: Skin Right Forearm, R/O Rhus Derm

117

Clinical Figure(s) Clinical Differential Diagnosis • • • •

Allergic contact dermatitis Atopic Dermatitis Drug reaction Arthropod bite reaction

Histopathology Sections of this punch biopsy of skin reveal superficial and deep perivascular and interstitial mixed inflammatory infiltrate with lymphocytes, abundant eosinophils and occasional neutrophils with spongiotic intraepidermal vesicles (Figs. 18.2

Figure 18.2  Panoramic view of the section, low magnification

118

D.-L. Xie et al.

Figure 18.3  One area with marked inflammation centered on bright red material, “flame figures”, medium magnification

and 18.3). Eosinophil degranulation onto collagen fibers forming “flame figures” is identified (Figs. 18.4 and 18.5). The overlying epidermis has spongiosis. PAS stain is negative for fungal organisms.

Diagnosis –– Eosinophilic cellulitis (Well’s syndrome).

Case Discussion • Eosinophilic cellulitis (Well’s syndrome) is a pattern diagnosis and can be seen in variety of conditions, including:

Chapter 18.  Case 18: Skin Right Forearm, R/O Rhus Derm

119

Figure 18.4 Numerous eosinophils are among the inflammatory cells, medium high magnification

Figure 18.5  Bright red area, “flame figure”, and numerous eosinophils, medium high magnification

120

–– –– –– –– ––

D.-L. Xie et al.

Arthropod bite reaction. Drug reaction. Infection and infestation. Non-specific allergic/atopic dermatitis. Some patients may have an associated underlying hematologic disorder.

Clinical Management Based on histopathological and clinical correlation, a diagnosis of Well’s syndrome was made. The patient was prescribed antihistamines, topical clobetasol cream, and was advised to finish her steroid taper.

Suggested Reading Coelho de Sousa V, Laureano Oliveira A, Cardoso J. Successful treatment of eosinophilic cellulitis with dapsone. Dermatol Online J. 2016;22(7):13030/qt9v67b10b. Newton JA, Greaves MW.  Eosinophilic cellulitis (Well’s syndrome) with florid histological changes. Clin Exp Dermatol. 1988;13(5):318–20. Panizzon R.  Well’s syndrome (eosinophilic cellulitis): additional cases in the literature. J Am Acad Dermatol. 1989;20(6):1136–7. Peckruhn M, Tittelbach J, Schliemann S, Elsner P. Life of lesions in eosinophilic cellulitis (Well’s syndrome)- a condition that may be missed at first sight. Am J Dermatopathol. 2015;37(2):e15–7.

Chapter 19 Case 19: Left Mandible Skin, R/O Basal Cell Carcinoma vs. Cyst Dong-Lin Xie, Tam H. Nguyen, and Robert A. Norman

History and Clinical A 47-year-old female presented for evaluation of a mole on her left jawline for the past 7 years. She stated that it had not grown significantly over the years and denied any pain or pruritis. She had no personal history of skin cancer.

Physical Examination The patient had a solitary, smooth, skin-colored, dome-­ shaped papule on the left mandibular region.

D.-L. Xie (*) Ruffolo, Hooper & Associates, M.D, P.A., Tampa, FL, USA e-mail: [email protected] T. H. Nguyen DO, PGY-1 Larkin Community Hospital, Miami, FL, USA R. A. Norman Department of Dermatology, Nova Southeastern University, Tampa, FL, USA Center for Geriatric Dermatology and Neuro-Dermatology, Tampa, FL, USA © Springer Nature Switzerland AG 2020 D.-L. Xie (ed.), Clinical Cases in Dermatopathology, Clinical Cases in Dermatology, https://doi.org/10.1007/978-3-030-28807-5_19

121

122

D.-L. Xie et al.

Clinical Differential Diagnosis • • • •

Angiofibroma Dermal nevus Syringoma Sebaceous adenoma

Histology and Diagnosis Histopathology Sections of this punch biopsy reveal a dermal proliferation of cords and nests of basaloid epithelial cells with clear cytoplasm in a fibrotic stroma (Figs. 19.1 and 19.2). Some basaloid buds show follicular infundibular differentiation. Comedone-­ like horn cysts are present, some of which ruptured with adjacent granulomatous inflammation (Figs.  19.2 and 19.3). Mitoses are present.

Figure 19.1  Panoramic view of the lesion, low magnification

Chapter 19.  Case 19: Left Mandible Skin, R/O Basal…

123

Figure 19.2 Clear cells, horn cysts and granulomatous inflammation, medium low magnification

The clear cell cytoplasm is negative with PAS/diastase, consistent with glycogen in the cytoplasm, which suggests outer root sheath differentiation. Immuno studies reveal that the tumor cells are positive CK5/6 (Fig. 19.4), p40 (Fig. 19.5) and partially positive for p16 (Fig.  19.6). Histologic section from the area with positive p16 stain reveals that the nuclei are hyperchromatic (Fig.  19.7). Tumor cells are essentially negative for CEA, BerEP4, CK7, EMA and CAM5.2.

Diagnosis –– Consistent with trichoepithelioma with clear cell changes and atypical features.

Case Discussion • This trichoepithelioma has unusual histologic features. At first glance, it looks like clear cell syringoma. However,

124

D.-L. Xie et al.

Figure 19.3 Clear cells, comedone-like horn cysts and basaloid nests and cords, medium low magnification

Figure 19.4  Immunohistochemical stain for CK5/6 is positive

Chapter 19.  Case 19: Left Mandible Skin, R/O Basal…

125

Figure 19.5 Immunohistochemical stain for p40 is positive with nuclear stain

Figure 19.6  Immunohistochemical stain for p16 is partially positive in some area

126

D.-L. Xie et al.

Figure 19.7 Histologic section from the area with positive p16 reveals that the nuclei are hyperchromatic, medium high magnification

immunologic findings and PAS/diastase are consistent with follicular differentiation. • Trichoepithelioma with clear cell cytoplasm is uncommon. This lesion also has atypical features, including partial p16 positive, mitoses and hyperchromatic nuclei, and complete but conservative removal of the residual lesion was recommended. • Histological differential considerations include: –– Clear cell syringoma. –– Basal cell carcinoma with clear cell changes. –– Other eccrine and apocrine adnexal neoplasms with clear cell changes. • Immuno histochemical studies performed in this case will help to eliminate most of the above considerations.

Chapter 19.  Case 19: Left Mandible Skin, R/O Basal…

127

Clinical Management Based on histopathological and clinical correlation, a diagnosis of a trichoepithelioma was made. The patient was referred to a plastic surgeon for removal of the lesion.

Suggested Reading Gupta A, Ali MJ, Mishra DK, Naik MN. Solitary trichoepithelioma of the eyelid: a clinico-pathological correlation. Int J Trichology. 2015;7(2):80–1. Mohammadi AA, Seyed Jafari SM.  Trichoepithelioma: a rare but crucial dermatologic issue. World J Plast Surg. 2014;3(2):142–5. Teli B, Thrishuli PB, Santhosh R, Amar DN, Rajpurohit S. Giant solitary trichoepithelioma. South Asian J Cancer. 2015;4(1):41–4.

Chapter 20 Case 20: Occipital Scalp Skin, R/O Squamous Cell Carcinoma Dong-Lin Xie, Tam H. Nguyen, and Robert A. Norman

History and Clinical A 35-year-old female with a history of acne and eczema presented with an irregular growth on the back of her neck for the past several years. She reported that the growth was very itchy and bled a lot after scratching. There was no personal history of skin cancer.

D.-L. Xie (*) Ruffolo, Hooper & Associates, M.D, P.A., Tampa, FL, USA e-mail: [email protected] T. H. Nguyen DO, PGY-1 Larkin Community Hospital, Miami, FL, USA R. A. Norman Department of Dermatology, Nova Southeastern University, Tampa, FL, USA Center for Geriatric Dermatology and Neuro-Dermatology, Tampa, FL, USA © Springer Nature Switzerland AG 2020 D.-L. Xie (ed.), Clinical Cases in Dermatopathology, Clinical Cases in Dermatology, https://doi.org/10.1007/978-3-030-28807-5_20

129

130

D.-L. Xie et al.

Figure 20.1  Discrete, erythematous verrucoid papule with areas of erosion

Physical Examination The patient had a 1.5  cm, discrete, erythematous verrucous papule with areas of erosion on the occipital scalp (Figs. 20.1 and 20.2).

Clinical Figures Clinical Differential Diagnosis • • • • •

Keratoacanthoma Angiokeratoma Pyogenic granuloma Irritated seborrheic keratosis Hemangioma

Chapter 20.  Case 20: Occipital Scalp Skin, R/O…

131

Figure 20.2  Closer look of this erythematous verrucoid papule with areas of erosion

Histopathology Sections of this punch biopsy reveal a dome to polypoid dermal lesion with proliferation of lobulated vessels and marked inflammation (Fig. 20.3). The marked inflammation is mostly superficially located (Fig. 20.4). Aggregated thick-walled and thin-walled vessels lined by flattened endothelial cells are beneath the inflammation (Fig.  20.5). Mixed inflammatory cells of lymphocytes, plasma cells and eosinophils are identified (Fig. 20.6). Under higher magnification in the markedly inflamed area, there is a background vascular proliferation with plumped epithelioid endothelial cells with abundant eosinophils (Fig. 20.7).

132

D.-L. Xie et al.

Figure 20.3  Panoramic view of lobulated vascular proliferation and marked inflammation, low magnification

Figure 20.4 Superficially located marked inflammation, medium low magnification

Chapter 20.  Case 20: Occipital Scalp Skin, R/O…

133

Figure 20.5 Aggregated thick-walled and thin-walled vessels, medium magnification

Figure 20.6  Mixed inflammatory cells, mostly lymphocytes, medium magnification

134

D.-L. Xie et al.

Figure 20.7  Capillary vessels with plumped epithelioid endothelial cells and abundant eosinophils, medium high magnification

Final Diagnosis –– Favored to hemangioma.

represent

inflamed

arterio-venous

Case Discussion • This is not a garden variety of arterio-venous hemangioma. It has moderate to dense inflammatory infiltrate in the superficial part of the lesion with frequent eosinophils. The endothelial cells are somewhat epithelioid. The vascular proliferation has a lobular pattern. • Pyogenic granuloma is one histologic differential consideration. • Angiolymphoid hyperplasia with eosinophils (ALHE) is another histologic differential consideration, which is generally considered as a reactive lymphoid hyperplasia.

Chapter 20.  Case 20: Occipital Scalp Skin, R/O…

135

Clinical Management Based on histopathological and clinical correlation, a diagnosis of an inflamed arterio-venous hemangioma was made. The patient was reassured about the benign nature of the lesion and was advised to follow up as needed.

Suggested Reading Clairwood MQ, Bruckner AL, Dadras SS. Verrucous hemangioma: a report of two cases and review of the literature. J Cutan Pathol. 2011;38(9):740–6. Eschard C. Infantile haemangioma: When investigation is necessary and current therapeutic developments. Ann Dermatol Venereol. 2015;142(8–9):476–82. Lara-Corrales I, Somers GR, Ho N. Verrucous hemangioma: a challenging vascular lesion. J Cutan Med Surg. 2010;14(3):144–6.

Chapter 21 Case 21: Skin from Mid-­Forehead, R/O Verruca Vulgaris Dong-Lin Xie, Tam H. Nguyen, and Robert A. Norman

History and Clinical An 11-year-old female presented with a “strawberry mark” on her forehead since birth. Over the years, the mass grew slowly, and “spiky” projections emerged from the base. The patient denied any pain, pruritis, or bleeding from the area, but noted it was cosmetically concerning.

Physical Examination The patient had yellow to orange colored papules coalescing into a large annular shaped plaque on the midline of the fore-

D.-L. Xie (*) Ruffolo, Hooper & Associates, M.D, P.A., Tampa, FL, USA e-mail: [email protected] T. H. Nguyen DO, PGY-1 Larkin Community Hospital, Miami, FL, USA R. A. Norman Department of Dermatology, Nova Southeastern University, Tampa, FL, USA Center for Geriatric Dermatology and Neuro-Dermatology, Tampa, FL, USA © Springer Nature Switzerland AG 2020 D.-L. Xie (ed.), Clinical Cases in Dermatopathology, Clinical Cases in Dermatology, https://doi.org/10.1007/978-3-030-28807-5_21

137

138

D.-L. Xie et al.

Figure 21.1 Slightly yellow to orange colored papules coalescing into a large annular shaped plaque

head (Fig.  21.1). There were overlying cutaneous spicules emerging from the plaque upon closer examination.

Clinical Figure(s) Clinical Differential Diagnosis • • • •

Nevus sebaceus Juvenile xanthogranuloma Verruca vulgaris Congenital nevus

Chapter 21.  Case 21: Skin from Mid-Forehead, R/O…

139

Figure 21.2  Panoramic view of marked papillomatosis, low magnification

Histopathology Sections of this shave biopsy of skin reveal hyperkeratosis, marked papillomatosis and acanthosis (Fig. 21.2). Coarse round keratohyalin granules and vacuolization of upper epidermal cells are found (Fig. 21.3). Underlying sebaceous proliferation with sebaceous glands having disorganized appearance is identified (Figs.  21.4 and 21.5). Some of the sebaceous ducts have direct opening to the surface (Fig. 21.6).

Final Diagnosis –– Combined verruca vulgaris and nevus sebaceus of Jadassohn.

Pathology Discussion • Most cases of nevus sebaceus of Jadassohn have verrucous epidermal hyperplasia. This lesion may entirely represent

140

D.-L. Xie et al.

Figure 21.3 Coarse keratohyalin granules and vacuolization of upper epidermal cells, medium low magnification

Figure 21.4 Underlying disorganized sebaceous proliferation, medium low magnification

Chapter 21.  Case 21: Skin from Mid-Forehead, R/O…

141

Figure 21.5 Another area of underlying disorganized sebaceous proliferation, medium low magnification

Figure 21.6  Sebaceous duct directly opens to the surface, medium low magnification

142

D.-L. Xie et al.

nevus sebaceus of Jadassohn with exuberant proliferation of papillomatosis. • However, verrucous epidermal hyperplasia to this prominence is a more typical feature of verruca and is uncommon in a garden variety of nevus sebaceus of Jadassohn. The findings in this lesion are favored to represent a combined verruca vulgaris and nevus sebaceus of Jadassohn. • Other lesions may arise in nevus sebaceus of Jadassohn, including epidermal nevus, basal cell carcinoma, and syringocystadenoma papilliferum.

Clinical Management Based on histopathological and clinical correlation, a diagnosis of a combined verruca vulgaris and nevus sebaceus of Jadassohn was made. The patient was advised about the condition, and the increased risk of basal cell carcinomas within the lesion. The patient’s nevus sebaceus of Jadassohn was treated with a complete shave excision, and margins were assessed.

Suggested Reading Chepla KJ, Gosain AK.  Giant nevus sebaceus: definition, surgical techniques, and rationale for treatment. Plast Reconstr Surg. 2012;130:296e–304e. Cicek AF, Aykan A, Yapici A, Gamsizkan M, Ozturk S, Demiriz M. Nevus sebaceus with basal cell carcinoma, poroma, and verruca vulgaris. Indian J Pathol Microbiol. 2015;58(4):534–6. Gozel S, Donmez M, Akdur NC, Yikilkan H.  Development of six tumors in a sebaceus nevus of Jadassohn: report of a case. Korean J Pathol. 2013;47(6):569–74. Heilig S, Koslosky K, Loffreda MD, Shin HT, Zaenglein AL. Eczematous nevus sebaceus: a report of three cases. Pediatr Dermatol. 2011;28(2):176–9. Idriss MH, Elston DM. Secondary neoplasms associated with nevus sebaceus of Jadassohn: a study of 707 cases. J Am Acad Dermatol. 2014;70(2):332–7. Tadlaoui I, Benzekri L, Afifi Y, Mrabti H, Azib N, Mazzouz S, Belmahi A, Kettani F, Hassam B. Trichoblastoma in Jadassohn’s nevus sebaceus. Ann Dermatol Venereol. 2007;134(3 Pt 1):300–1.

Chapter 22 Case 22: Right Upper Arm Lesion Dong-Lin Xie, Emelia Farnsworth, and Robert A. Norman

History and Clinical A 45 year old male with a history of herpes simplex, folliculitis, HIV, testicular, arm, and buttock lesions, and squamous cell carcinoma presented with a large, hypertrophic lesion on the right upper arm. Patient reported pruritus and pain associated with the site. According to the patient, the neoplasm had been present for 3 weeks and was slowly becoming larger.

D.-L. Xie (*) Ruffolo, Hooper & Associates, M.D, P.A., Tampa, FL, USA e-mail: [email protected] E. Farnsworth MS1, Nova Southeastern University, Tampa, FL, USA R. A. Norman Department of Dermatology, Nova Southeastern University, Tampa, FL, USA Center for Geriatric Dermatology and Neuro-Dermatology, Tampa, FL, USA © Springer Nature Switzerland AG 2020 D.-L. Xie (ed.), Clinical Cases in Dermatopathology, Clinical Cases in Dermatology, https://doi.org/10.1007/978-3-030-28807-5_22

143

144

D.-L. Xie et al.

Physical Examination The left malar crescent was raised, verrucoid, and thickened. The lesion was bosselated, and a grey-white color. A 7.0 × 6.0 × 1.0 cm sample was collected in eight pieces by excisional biopsy and sent to pathology. The lesion itself was 20 mm in diameter. The initial biopsy from the same location two weeks before was a shave of 0.8 × 0.5 × 0.1 cm specimen.

Clinical Differential Diagnosis • • • • •

Squamous Cell Carcinoma Atypical Mycobacterial Infection Herpes Simplex Chromomycosis Granuloma Inguinale

Histopathology and Diagnosis Histopathology The specimen from the initial biopsy (Fig.  22.1) revealed mixed with acute and chronic inflammation accompanied by cells with ground glass nuclei and margination of nuclear material with multinucleate epithelial cells, which is typical viral cytopathic changes for herpetic cells. The second specimen from two weeks later reveals marked proliferation of squamous epithelium, i.e. pseudoepitheliomatous hyperplasia (PEH) with mild reactive squamous atypia (Figs.  22.2 and 22.3). Inflammation and fibrosis are present in the stroma. Among the inflammatory cells are ­frequent plasma cells, lymphocytes, and eosinophils. Hair follicles had been destroyed, but in some areas the hair follicles were dilated with many acantholytic cells. AFB (Fig.  22.4), GMS (Fig. 22.5), Fite stains, and tissue Gram stains were performed and found to be negative. The Steiner Stain (Fig. 22.6),

Chapter 22.  Case 22: Right Upper Arm Lesion

145

Figure 22.1  Viral cytopathic changes with acute and chronic inflammation accompanied by acantholytic keratinocytes with ground glass nuclei, margination of nuclear material and multinucleate epithelial cells, medium high magnification

Figure 22.2  2.5×: Marked proliferation of squamous epithelium, i.e. pseudoepitheliomatous hyperplasia (PEH) with mild reactive squamous atypia, low magnification

146

D.-L. Xie et al.

Figure 22.3 2.5×: Another field with PEH and dilated follicular infundibulum, low magnification

Figure 22.4  AFB stain is negative for mycobacteria

Chapter 22.  Case 22: Right Upper Arm Lesion

147

Figure 22.5  GMS stain is negative for fungal organisms

Figure 22.6  Stiner stain has some suspicious stain in the epidermis but is inconclusive

148

D.-L. Xie et al.

however, demonstrated that there were likely organisms within the epidermis but is inclusive.

Final Diagnosis –– HIV/AIDS. –– Herpetic viral infection of the initial biopsy. –– Pseudoepitheliomatous hyperplasia.

Case Discussion • Pseudoepitheliomatous Hyperplasia (PEH) is a benign reactive and reparative process, which can be seen in chronic irritation and inflammation, damage to the epithelium trough trauma or surgery, long-standing infection from bacteria or fungi, non-healing leg ulcer in venous insufficiency and diabetic ulcer and reaction to underlying tumor. • In this case, PEH was a reparative and regenerative reaction to the prior biopsy of a lesion, which was caused by herpetic viral infection. • The histologic features of herpetic viral infection caused by herpes simplex (HSV) or varicella zoster (shingles) are similar. The location of the herpetic lesions can provide clues of HSV vs. VZV.  However, in a immunodeficient patient, the infection may occur in uncommon locations. Immuno histochemical studies are the best way to differentiate HSV and VZV. • Immunodeficient patients, including HIV/AIDS patients, patients under chemotherapy and immune suppressive therapy, are susceptible to multiple infections. Thorough evaluation of suspicious lesion, which includes AFB, GMS, Fite stains as well as immuno studies. Today, immuno stain for spirochete has replaced Steiner’s stain. • Sometimes, it can be extremely difficult to histologically distinguish PEH from squamous cell carcinoma, especially if the specimen only has superficial portion of the lesion.

Chapter 22.  Case 22: Right Upper Arm Lesion

149

• The most typical underlying tumor that has overlying PEH is granular cell tumor. One of the common fungal infection that cause PEH is blastomycosis. In both situations, PEH can be mistaken as well differentiated squamous cell carcinoma.

Clinical Management PEH is benign and can be managed with conservative surgical excision and antibiotics. After removing the lesion, the patient was treated with Valtrex 500  mg once daily for 30  days and zovirax 5% ointment to apply to the lesion six times daily for 7  days. The patient’s herpetic infection was markedly improved.

Suggested Reading Chakrabarti S, Chakrabarti P, Agrawal D, Somanath S. Pseudoepitheliomatous hyperplasia: a clinical entity mistaken for squamous cell carcinoma. J Cutan Aesthet Surg. 2014;7(4):232. https://doi.org/10.4103/0974-2077.150787. Elwood H., Christopher H. Skin—Nonmelanocytic tumors & other tumors of skin: dermatopathology patterns. Pathology Outlines. 2012. http://www.pathologyoutlines.com/topic/skintumornonmelanocyticpatterns.html. Accessed 22 Oct 2018. Ling Y, Zhu C, Wen S, Luo R, Li P, Cao Y, Rao H, Lin S, Cai M. Pseudoepitheliomatous hyperplasia mimicking invasive squamous cell carcinoma in extranodal natural killer/T-cell lymphoma: a report of 34 cases. Histopathology. 2015;67(3):404–9. https://doi. org/10.1111/his.12656. Mcfaddin C, Sirohi D, Castro-Echeverry E, Fernandez MP.  Desmoplastic trichoepithelioma with pseudocarcinomatous hyperplasia: a report of three cases. J Cutan Pathol. 2014;42(2):102–7. https://doi.org/10.1111/cup.12442.

Chapter 23 Case 23: Right Arm Papules Dong-Lin Xie, Emelia Farnsworth, and Robert A. Norman

History and Clinical A 65  year old female with history of multicentric invasive ductal breast carcinoma with lobular features and asthma presented with a chief complaint of lesions on the right arm. The lesions were visualized during examination, and patient noted no symptoms such as pruritus or bleeding associated with them.

D.-L. Xie (*) Ruffolo, Hooper & Associates, M.D, P.A., Tampa, FL, USA e-mail: [email protected] E. Farnsworth MS1, Nova Southeastern University, Tampa, FL, USA R. A. Norman Department of Dermatology, Nova Southeastern University, Tampa, FL, USA Center for Geriatric Dermatology and Neuro-Dermatology, Tampa, FL, USA © Springer Nature Switzerland AG 2020 D.-L. Xie (ed.), Clinical Cases in Dermatopathology, Clinical Cases in Dermatology, https://doi.org/10.1007/978-3-030-28807-5_23

151

152

D.-L. Xie et al.

Physical Examination Mildly erythematous papules were present on the patient’s right axilla (Fig.  23.1) and right upper arm/shoulder areas (Fig.  23.2). The lesions are raised. The patient has an additional lesion on the right forearm which has scaly appearance. An 8 mm punch biopsy to the depth of 1.1 cm was obtained from the patient right upper arm/shoulder area. A shave biopsy was obtained from the right forearm, which measured 0.5 × 0.4 × 0.1 cm.

Figure 23.1 Lesions in right axilla, mildly erythematous elevated papules

Chapter 23.  Case 23: Right Arm Papules

153

Figure 23.2 Similar lesions on right upper arm/shoulder, mildly erythematous elevated papules

Clinical Figures Clinical Differential Diagnosis • • • • •

Squamous Cell Carcinoma Basal Cell Carcinoma Malignant Melanoma Actinic Keratosis Bowen’s Disease

Histopathology The first biopsy from the right upper arm/shoulder area reveal a diffuse infiltrative tumor cells between collagen bundles through the entire reticular dermis at different magnification (Figs. 23.3, 23.4, 23.5, and 23.6). The tumor cells are clustered in small nests and single cell strands in an “Indian-­ file” pattern (Fig. 23.6).

154

D.-L. Xie et al.

Figure 23.3  Panoramic view of a diffuse dermal process

Figure 23.4  Low magnification showing a diffuse infiltrative dermal process

Chapter 23.  Case 23: Right Arm Papules

155

Figure 23.5  Medial low magnification showing a diffuse neoplastic infiltration between collagen bundles

Figure 23.6 High magnification showing malignant tumor cells are clustered in small nests and single cell strands in an “Indian-file” pattern

156

D.-L. Xie et al.

It was noted that the neoplasm cell stained positive with pan-cytokeratin (Fig. 23.7), CK7 (Fig. 23.8), estrogen receptor (Fig.  23.9), focally positive with progesterone receptor (Fig.  23.10) and negative with S-100 stain (Fig.  23.11). The immuno-stain for Her2/Neu was negative in the tumor cells (Fig. 23.12). The second biopsy from the forearm (Fig.  23.13) reveals dysplastic squamous cells through the entire thickness of epidermis, which continues laterally with background hypertrophic actinic keratosis. Cutaneous horn on the surface is present. The lesion is transected at the base. Invasion cannot be entirely excluded.

Final Diagnosis –– Right upper arm/shoulder lesion: Metastatic breast carcinoma, favoring lobular type. –– Right forearm lesion: Squamous cell carcinoma in situ in background of actinic keratosis.

Figure 23.7 Immuno histochemical stain for Pan Keratin is diffusely positive in tumor cells

Chapter 23.  Case 23: Right Arm Papules

157

Figure 23.8  Immuno histochemical stain for CK7 is diffusely positive in tumor cells

Figure 23.9  Immuno histochemical stain for ER (estrogen receptor) is diffusely positive in tumor cells with nuclear stain

158

D.-L. Xie et al.

Figure 23.10 Immuno histochemical stain for PR (progesterone receptor) is positive in some tumor cells with nuclear stain

Figure 23.11  Immuno histochemical stain for S-100 is negative in tumor cells

Chapter 23.  Case 23: Right Arm Papules

159

Figure 23.12  Immuno histochemical stain for Her2/Neu is negative in tumor cells

Figure 23.13  Biopsy of the forearm lesion: full thickness of dysplastic squamous cells (left part in the picture) with cutaneous horn on the surface (tall stack of pink material). Background hypertrophic actinic keratosis (right part in the picture) is present

160

D.-L. Xie et al.

Case Discussion • Skin metastases can mimic other dermatologic conditions, including dermatitis, cellulitis and cutaneous mucinosis. Clinical consideration of possible metastases depends on thorough inquiring of patient’s past medical history. More common than not in these metastatic lesions, the frequent clinical histories associated with the biopsy are skin rash. And the diagnosis of metastasis becomes a surprise. • Though relatively uncommon, the underlying primary malignancies of cutaneous metastases are breast carcinoma, gastric carcinoma, prostate carcinoma and renal cell carcinoma among others. Immuno histochemical studies are necessary to help to identify or confirm the possible primary sites. Pertinent past history will help to select immuno studies and possibly with fewer immuno stains to reach the diagnosis. • Cutaneous metastases of breast carcinoma may represent contiguous extension of the original breast carcinoma, local metastasis or distant metastasis. Both ductal and lobular breast carcinoma can have cutaneous metastases. Correlation with the original breast carcinoma is indicated. The immuno profile may change in the surviving metastatic tumor cells, which may impact treatment options. • Multiple erythematous lesions at the presentation may give a clinical impression of rashes of similar process. However, they may represent different conditions. B ­ iopsies of more than one lesion may be necessary. In this patient, the right forearm lesion has a scaly appearance. The biopsy of this forearm lesion reveals squamous cell carcinoma insitu, arising in a background of hypertrophic actinic keratosis. Since this lesion is transected at the base, invasion cannot be excluded. A deeper biopsy may be needed if the lesion persists. Squamous cell carcinoma, basal cell carcinoma and melanoma are common skin cancers on sunexposed skin.

Chapter 23.  Case 23: Right Arm Papules

161

Clinical Management Patient was instructed to follow up with her oncology team.

Suggested Reading Lambert SR, Mladkova N, Gulati A, Hamoudi R, Purdie K, Cerio R, Leigh I, Proby C, Harwood CA.  Key differences identified between actinic keratosis and cutaneous squamous cell carcinoma by transcriptome profiling. Br J Cancer. 2014;110(2):520–9. https://doi.org/10.1038/bjc.2013.760. Martin H, Mariano-Bourin M, Antunes L, Bonhomme A, Cuny JF, Dubois L, Truchetet F, Schoffler A. Cutaneous metastases on the extremities. Ann Dermatol Venereol. 2019;146(2):115–20. https:// doi.org/10.1016/j.annder.2018.07.025. Potenza C, Bernardini N, Balduzzi V, Losco L, Mambrin A, Marchesiello A, Tolino E, Zuber S, Skroza N, Proietti I. A review of the literature of surgical and nonsurgical treatments of invasive squamous cells carcinoma. Biomed Res Int. 2018;2018:1–9. https://doi.org/10.1155/2018/9489163. Sehrawat A, Kotwal S, Parthasarathy K.  Metastatic dermatosis in breast carcinoma on adjuvant trastuzumab: is skin a sanctuary site in human epidermal growth factor receptor-2-amplified ­disease? Indian J Dermatol. 2018;63(5):424. https://doi.org/10.4103/ijd. ijd_234_18.

Chapter 24 Case 24: Nodular Masses on Left Fingertips Dong-Lin Xie, Emelia Farnsworth, and Robert A. Norman

History and Clinical A 54 y/o male presented to clinic with elevated lesions on his left first, second, and third fingertips. He reports he had orthopedic hand surgery 2  weeks prior on the left hand. Sutures were still present. Patient reports a history of acne, otitis externa, and xerosis of skin.

D.-L. Xie (*) Ruffolo, Hooper & Associates, M.D, P.A., Tampa, FL, USA e-mail: [email protected] E. Farnsworth MS1, Nova Southeastern University, Tampa, FL, USA R. A. Norman Department of Dermatology, Nova Southeastern University, Tampa, FL, USA Center for Geriatric Dermatology and Neuro-Dermatology, Tampa, FL, USA © Springer Nature Switzerland AG 2020 D.-L. Xie (ed.), Clinical Cases in Dermatopathology, Clinical Cases in Dermatology, https://doi.org/10.1007/978-3-030-28807-5_24

163

164

D.-L. Xie et al.

Physical Examination Multiple erythematous somewhat fused elevated nodular appearing lesions on his thumb and other fingers (Fig.  24.1, thumb lesions). Three biopsies were obtained from superior, medial and lateral lesions. The biopsy from the superior aspect of the left thumb was submitted for direct immunofluorescent studies. No discharge was noted during the biopsy procedure, however the skin appears dry and slightly peeling.

Clinical Figure(s) Clinical Differential Diagnosis • • • • •

Allergic Contact Dermatitis Bullous Pemphigoid Granulomas Urticaria Linear IgA dermatosis

Figure 24.1  Elevated erythematous lesions of the left thumb

Chapter 24.  Case 24: Nodular Masses on Left Fingertips

165

Histopathology The biopsy from the lateral aspect of thumb that was submitted for regular H&E is a superficial shave specimen which has focal epithelioid granuloma in the papillary dermis (Fig. 24.2). No polarizable foreign material is identified. PAS stain is negative for fungal organisms. Since the lesion is very focal in this biopsy, no additional studies could be performed. The biopsy from the medial thumb is superficial without underlying pathological tissue present. The specimen from superior portion that was submitted for the direct immunofluorescent studies reveals negative for IgG, IgM, IgA, and C3, i.e., negative for immunobullous disorder. However, additional frozen sections stained with H&E from this biopsy (frozen sections are somewhat lack of cytologic details: Figs.  24.3, 24.4, and 24.5) reveal fused nests of large epithelioid cells. The epithelioid cells were positive for CD68 (Fig. 24.6) and negative for pankeratin, CK5/6, P40, S-100, and Mart-1. The immuno studies support that the ­epithelioid cells

Figure 24.2 Focal epithelioid granuloma in the papillary dermis. Note the cornified layer from acral skin is very thick

166

D.-L. Xie et al.

Figure 24.3  Panoramic view of fused nests of cells, low magnification

Figure 24.4  Closer look of fused nests/nodules of epithelioid cells, low magnification

Chapter 24.  Case 24: Nodular Masses on Left Fingertips

167

Figure 24.5  High magnification with a more detailed view of hyperchromatic epithelioid cell aggregates rimed with lymphocytes

Figure 24.6  Immuno stain for CD68 is positive in the epithelioid cells

168

D.-L. Xie et al.

are histiocytes, consistent with epithelioid granulomas. Immuno studies also exclude neoplasm. The AFB (Fig. 24.7) and GMS (Fig. 24.8) stains were negative for microorganisms. No polarizable foreign material is identified.

Final Diagnosis –– Epithelioid granulomas, most suggestive of sarcoidal granulomas.

Case Discussion • Granulomatous inflammation can be triggered by a variety of insults, including infections, foreign body reaction, autoimmune disorders, and adverse reactions to drugs as well as ruptured folliculitis among other conditions. Sarcoidal granuloma/sarcoidosis is considered an autoimmune type of granuloma.

Figure 24.7  AFB is negative for mycobacteria

Chapter 24.  Case 24: Nodular Masses on Left Fingertips

169

Figure 24.8  GMS is negative for fungal organisms

• Histologically, traditional granulomas may have features of necrotizing or caseating granuloma, foreign body type granuloma with many giant cells, suppurative granuloma and epithelioid granulomas. Granuloma annulare and necrobiosis lipoidica are also granulomatous inflammations, but are not classified as traditional granulomas. • Necrotizing granulomas or caseating granulomas are often seen in tuberculosis. Most of the fungal infection related granulomas are non-necrotizing granulomas. Suppurative granulomas are frequently seen in ruptured folliculitis. Ruptured cyst usually has histologic features of foreign body type granulomatous inflammation and frequently has combined features of suppurative and foreign body type granulomatous inflammation. Sarcoidosis is a type nonnecrotizing fused epithelioid granulomatous inflammation. • In sarcoidal granuloma, as in most of the other granulomatous inflammation, exclusion of infection and foreign material are indicated. AFB and GMS stains are generally performed. Reviewing the sections under polarized microscope to exclude foreign material is indicated.

170

D.-L. Xie et al.

• Sarcoidal granulomas of skin may be localized. However, systemic involvement can happen in some patients. Chest X-ray to exclude pulmonary involvement may be considered. In addition, the ratio of T helper and T cytotoxic lymphocytes may change. • In this patient, the biopsies are from acral skin. Acral skin has thick cornified/keratin layer, and frequently thicker keratin layer in diseased acral skin. A one millimeter thick shave biopsy can still be too superficial to have underlying lesion even does not appear so clinically. Though the two biopsies submitted for histology examination were superficial and without enough lesional tissue for additional studies, the specimen submitted for the direct immunofluorescent studies had remaining lesional tissue after direct immunofluorescent studies. Direct immunofluorescent studies were performed on frozen sections. However, H&E stained frozen sections in this case artificially made the epithelioid granulomas hyperchromatic, which could mislead to possible neoplasm. Thus, immuno histochemical studies were performed to exclude neoplasm.

Suggested Reading Esteves T, Tórtola T, Ferrer B, Aparicio G, Sulleiro E, González-­ López J, Moure Z, Romero D, Garcia-Patos V.  Use of molecular biology techniques in sarcoidal granulomatous dermatitis: a clinicopathological and molecular approach with diagnostic implications. Acta Derm Venereol. 2017;97(10):1241–2. https:// doi.org/10.2340/00015555-2745. Ishikawa M, Yamamoto T.  Subcutaneous sarcoidal granuloma underlying superficial basal cell carcinoma. Indian J Dermatol. 2015;60(6):638. https://doi.org/10.4103/0019-5154.159649. Shah KK, Pritt BS, Alexander MP. Histopathologic review of granulomatous inflammation. J Clin Tuberc Mycobact Dis. 2017;7:1–12. https://doi.org/10.1016/j.jctube.2017.02.001. Wanat KA, Rosenbach M. Cutaneous sarcoidosis. Clin Chest Med. 2015;36(4):685–702. https://doi.org/10.1016/j.ccm.2015.08.010.

Chapter 25 Case 25: Diffuse Rashes Across Back, Thighs, and Legs Dong-Lin Xie, Emelia Farnsworth, and Robert A. Norman

History and Clinical A 90  year-old male was seen in his senior living facility for evaluation of a diffuse rash across his back, thighs, and legs. The patient noted that the rash had been present for a few months and was extremely pruritic. Four shave biopsies were obtained of the rash, taken from the back, both thighs, and right leg.

D.-L. Xie (*) Ruffolo, Hooper & Associates, M.D, P.A., Tampa, FL, USA e-mail: [email protected] E. Farnsworth MS1, Nova Southeastern University, Tampa, FL, USA R. A. Norman Department of Dermatology, Nova Southeastern University, Tampa, FL, USA Center for Geriatric Dermatology and Neuro-Dermatology, Tampa, FL, USA © Springer Nature Switzerland AG 2020 D.-L. Xie (ed.), Clinical Cases in Dermatopathology, Clinical Cases in Dermatology, https://doi.org/10.1007/978-3-030-28807-5_25

171

172

D.-L. Xie et al.

Physical Examination Red to brown small discrete, elevated and waxy macules and papules papular lesions/rash were present. The maculopapular lesions are diffusely distributed across the patient’s back (Fig. 25.1) and bilateral lower extremities (Figs. 25.2 and 25.3). The greatest severity was on the back, with the rash’s intensity decreasing slightly on the extremities. A 0.7  ×  0.6  ×  0.1  cm shave biopsy was obtained from the patient’s back. Two shave biopsies, each measuring 0.5 × 0.3 × 0.1 cm and 0.5 × 0.4 × 0.1 cm were obtained from each thigh. One shave biopsy, measuring 0.6 × 0.5 × 0.1 cm was obtained from the patient’s right leg.

Clinical Figures Clinical Differential Diagnosis • • • • •

Vacuolar Dermatitis Drug Reaction Erythema Multiforme Bullous Pemphigoid Systemic Amyloidosis

Figure 25.1 Erythematous waxy and reticulated maculopapular rash on the back

Chapter 25.  Case 25: Diffuse Rashes Across Back…

173

Figure 25.2  Red to brown maculopapular rash on thigh

Figure 25.3 Red to brown slightly elevated waxy maculopapular rash on the shin

174

D.-L. Xie et al.

Histopathology and Diagnosis Histopathology All three biopsies have similar histologic features. The biopsy sections reveal dyskeratotic keratinocytes within the epidermis and interface vacuolar changes. This was accompanied with mild superficial inflammatory lymphocyte infiltrates. Pink globular clusters, mimicking “colloid bodies” were present mostly in the papillary dermis (Figs. 25.4, 25.5, and 25.6). More pink globular clusters are seen in the biopsy of the back and fewer in the biopsies of the thigh and the shin areas. The PAS stain was negative for fungal microorganisms. The Congo red stain reveals orange-pink clusters in the papillary dermis (Fig.  25.7), which correspond to the pink globular clusters in the histology sections. The orange-pink clusters of Congo red stained sections have “apple green” birefringence under polarized microscope (Fig. 25.8), consistent with amyloid material.

Figure 25.4  Pink “colloid bodies”, almost fused under lower magnification

Chapter 25.  Case 25: Diffuse Rashes Across Back…

175

Figure 25.5 Pink “colloid bodies”, the globular appearance under medium magnification

Figure 25.6  Pink “colloid bodies”, distinctive globular homogenized appearance under high magnification

176

D.-L. Xie et al.

Figure 25.7  The Congo red stain reveals orange-pink clusters in the superficial dermis, around small capillary vessels

Figure 25.8 The Congo red stained section reveal “apple green” birefringence under polarized microscope

Chapter 25.  Case 25: Diffuse Rashes Across Back…

177

Final Diagnosis –– Lichen and macular amyloidosis.

Pathology Discussion • Lichen and macular amyloidosis are considered variants of the same process and both forms may present in the same patient. Most of the time, lichen and macular amyloidosis are cutaneous, without visceral or systemic involvement. • The amyloid deposits are limited to papillary and superficial dermis. In macular amyloidosis, the deposits can be very small and easily overlooked. The material in lichen and macular amyloidosis is keratinocytic origin, and consists of tonofilaments among other components from degenerating keratinocytes discharged into dermis, where they are converted to amyloid. • The differential considerations of lichen and macular amyloidosis include primary systemic amyloidosis that involves skin, colloid milium, colloid bodies in lichen planus and other conditions. Clinical presentation, the distribution of the lesions, time of the onset and possible underlying conditions as well as histochemical studies may help to distinguish these conditions.

Clinical Management Patient was prescribed doxycycline 100 mg PO BID × 14 days as well as acidophilus PO BID × 14 days.

Suggested Reading Jayabhanu AA, Bubna AK, Rangarajan S, Veeraraghavan M, Joseph LD, Sundaram M. A clinicopathologic study of cutaneous amyloidosis at a tertiary health care center in South India. Pigment Int. 2016;3:17–23. Kibbi AG, Keibeiz NG, Zaynoun ST, Kurban AK. Primary localized cutaneous amyloidosis. Int J Dermatol. 1992;31(2):95–8. https:// doi.org/10.1111/j.1365-4362.1992.tb03245.x.

178

D.-L. Xie et al.

Tey HL, Cao T, Nattkemper LA, Tan VD, Pramono ZD, Yosipovitch G.  Pathophysiology of pruritus in primary localized cutaneous amyloidosis. Br J Dermatol. 2016;174(6):1345–50. https://doi. org/10.1111/bjd.14391. Wechalekar AD, Gillmore JD, Hawkins PM.  Systemic amyloidosis. Lancet. 2015;387(10038):2641–54. https://doi.org/10.1016/ S0140-6736(15)01274-X.

Index

A Abdomen, skin lesion, 49, 50, 52, 53 Acantholytic keratinocytes, 145 Acral skin, 170 Acrosyringium, 110–112 Adenocarcinoma, 81 Adipocytes in superficial dermis, 91 AFB stain negative for mycobacteria, 23 Alopecia on scalp, 41, 42 Angiolymphoid hyperplasia with eosinophils (ALHE), 134 Arterio-venous hemangioma, 133 B Basal cell carcinoma, 32 Basal keratinocytes, 37, 38 Borst-Jadassohn epithelioma, 107 Botox injection, 23 C Caseating granulomas, 169 Cellulitis, 160 Central follicle, 99

CK5/6, immunohistochemical stain, 124 Clear cells, 124 Clonal tumor cells, 107 Coarse keratohyalin granules, 140 Combined congenital melanocytic nevus and mesenchymal hamartoma, 93 Combined verruca vulgaris and nevus sebaceus of Jadassohn, 139, 142 Cutaneous metastases of breast carcinoma, 160 Cutaneous mucinosis, 160 D Demarcated clonal proliferation, 106 Dementia, 7 Dermal mixed inflammatory infiltrate of lymphocytes, histiocytes and multinucleated giant cells, 21 Dermatitis, 160 Dermatophytosis, 5

© Springer Nature Switzerland AG 2020 D.-L. Xie (ed.), Clinical Cases in Dermatopathology, Clinical Cases in Dermatology, https://doi.org/10.1007/978-3-030-28807-5

179

180

Index

Diffuse dermal process, 154 Diffuse infiltrative dermal process, 154 Diffuse neoplastic infiltration, 155 Direct immunofluorescent studies, 170 Discrete, erythematous verrucoid papule, 130 Disorganized follicular sebaceous elements, 91, 92 Disorganized sebaceous proliferation, 140 Dyskeratotic keratinocytes, 174 Dysplastic squamous cells, 159 Dysplastic/atypical intraepidermal poroma, 107 E Eosinophilic cellulitis (Well’s syndrome), 119 Epithelioid endothelial cells, 133, 134 Epithelioid granulomas, 168–170 Eruptive xanthoma, 59 Erythematous and scaly lesions on forehead, 27 Erythematous dermal nodule, 84 Erythematous lesions of left thumb, 164 Erythematous macules and papules on the forehead and temporal regions, 20 Erythematous plaque, 8 Erythematous verrucoid papule, 129, 130 Erythematous waxy and reticulated maculopapular rash on the back, 172 Estrogen receptor (ER), 80, 157 Extruding, 50

F Focal epithelioid granuloma in papillary dermis, 165 Forehead, skin lesions, 25, 26, 28–33 Foreign body suppurative granulomatous inflammation with phaeohyphomycosis, 87 Fungal hyphae, 4 Fungal organisms, GMS stain, 147 G Giant cells with foamy cytoplasm, 58 GMS stain negative for fungal organisms, 22 Granular cell tumor, 72, 74 Granular cytoplasm and larger pink droplets, 72 Granulomatous inflammation, 43, 168 Granulomatous reaction, 21, 23 H Her2/Neu, immuno histochemical stain, 159 Hidroacanthoma simplex, 107 Hyperchromatic epithelioid cell, 167 Hyperkeratosis and parakeratosis with neutrophils, 2 Hyperpigmented macules, 62 Hypopigmented macules, 36 I Immature radiating secondary follicles and fibrotic stroma, 100 Immature secondary follicles, 100

Index Immuno histochemical (IHC) studies, 9, 77–79, 81, 170 In situ hybridization (ISH) with kappa and lambda tests, 11 Interstitial and perivascular cells, 77 Interstitial cells, 77, 78 Intraepidermal poroma, 107 Invasive squamous cell carcinoma, 32 K Kappa ISH, 11 Kyrle’s disease, 53 L Lambda ISH, 12 Left lateral trunk lesions, 61, 63–65 Left mandible skin, 121, 123, 125, 126 Left parietal skin, 89, 90 Leg lesion with clinical impression, 7, 9–12 Lichen and macular amyloidosis, 177 M Maculopapular rash on shin and thigh, 173 Majocchi granuloma, 46 Malignant tumor cells, 155 Marked papillomatosis, 139 Melanin pigmentation, 39 Melanocytic cells, 92 Mesenchymal cells, 93 Metastatic breast carcinoma, 81, 82 Mid lateral back, skin, 97, 98 Midline parietal scalp, 90 Mildly erythematous papules, 152

181

Miliaria alba/ miliaria crystallina, 113 Miliaria pustulosa, 113 Miliaria rubra (prickly heat), 113 Muir-Torre syndrome, 32 Multicentric invasive ductal breast carcinoma, 151 Multiple erythematous lesions, 160 Multiple juvenile xanthogranulomas, 59 Multiple myeloma, 11, 12 Mycobacteria, AFB stain, 146 Myxoid stroma, 93 N Necrotizing granulomas, 169 Neutrophils in ductal epithelium, 112 Nevus sebaceus of Jadassohn, 139, 142 Nodular Masses on left fingertips, 163–165, 167, 168 O Occasional small eccrine ducts, 105 Occipital scalp skin, 129, 132–134 P p16, immunohistochemical stain, 125 p40, immunohistochemical stain, 125 Papillary dermis, colloid bodies, 174, 175 PAS positive for fungi, 87 PAS/diastase, 123 Perforating disorder, 53 Perforating folliculitis, 50 Phaeohyphomycosis, 87 Plasmacytoma, 11, 12 Polypoid dermal lesion, 131

182

Index

Porocarcinoma in situ, 108 Primary skin adnexal adenocarcinomas, 81 Progesterone receptor (PR), 80, 158 Prurigo nodularis, 53 Pseudoepitheliomatous hyperplasia (PEH), 144, 145, 148, 149 R Right arm papules, 151–153 Right inferior eyelid lesion, 55–58 Right upper arm/shoulder lesion, 160 S Sarcoidal granuloma, 169, 170 Sarcoidosis, 169 Scabies, 3–5, 17 Scalp, skin, 41, 42, 44–47 Sebaceoma, 28, 29 Sebaceous adenoma, 28, 29 Sebaceous carcinoma, 28, 30, 31 Sebaceous glands, 139 Sebaceous hyperplasia, 28, 30 Sebaceous proliferation, 141 Sebaceous tumors, 32 Skin lesions of trunk and extremities, 2, 5, 15–18 Skin right forearm, 115, 117, 119 Solitary juvenile xanthogranuloma, 59 Spongiosis and mixed inflammation, 3 Spongiotic epidermal changes, 16 Spongiotic intraepidermal vesicles, 117 Sporadic sebaceoma, 32 Squamous cell carcinomas, 32 Squamous epithelium, 145 Stasis vs. skin cancer, 7 Stratum corneum, 4

Superficial and mid dermal interstitial dermal infiltrate, 63, 64 Suppurative granuloma with foreign material, 85 Suppurative granulomatous infiltrate, 85 T Telangiectasia macularis eruptiva perstans (TMEP), 66 Tinea capitis with granulomatous inflammation (Majocchi granuloma), 45, 47 Touton giant cells, 56, 58 Transepidermal elimination of collagen and elastic fibers, 50 Trichoepithelioma, 123, 127 Trichofolliculoma, 101 Tumor cells, 123 U Ulcerated lesion with dense diffuse mononucleated cell infiltrate, 9 Ulcerated lesion with diffuse predominantly mononucleated cell infiltrate, 9 Upper abdomen, skin, 103, 104, 106 Upper arm, skin lesion, 69–74 Upper paraspinal lesion, 109, 111–113 Urticaria pigmentosa, 66 V Vague clonal proliferation, 105 Variable sized “Swiss cheese”like vacuoles, 22

Index Variable sized lesions on the lower back, 26 Verhoeff-Van Gieson (VVG) for elastic fibers, 50 Verrucous epidermal hyperplasia, 142 Vitiligo, 39

W Well’s syndrome, 120 X Xanthogranuloma, 59

183